Conference Notes 11-25-2015

Shorter version of Conference Notes this week.       Happy Thanksgiving!

 

Bonder/Gupta     Oral Boards

 

Case 1.  Pregnant woman presents with abdominal pain.  Patient was worked up with ultrasound and MRI.  Diagnosis was appendicitis.    The WBC count is less useful in pregnant patients due to baseline leukocytosis of pregnancy.

 

Case 2.  48 yo male with severe back pain and hypotension.   Patient has been taking ibuprofen, norco, and valium for pain without relief.   Patient has history of fever.   Physical exam demonstrated murmur. Diagnosis was spinal osteomyelitis/diskitis secondary to endocarditis.   Management was getting blood cultures, MRI of spine, echocardiogram and treating with IV antibiotics and consulting ID and Neurosurgery.  Harwood comment: Vancomycin and Gentamycin is the recommended empiric combination therapy for native valve endocarditis.

 

Case 3. 61 yo female with shortness of breath.   Patienthad normal CXR and markedly elevated d-dimer.  CTPE showed sub-massive PE.  Patient wastreated with heparin.

 

*Daniel Score >8 predicts worse outcome for PE.  More recent study by Shopp, Kline et al.  Says HR>100, S1Q3T3, Complete RBBB, Invert T waves V1-4, ST Elevation in AVR, and Afib are all independent predictors of increased risk of shock and death.


Harwood comment: No need to give O2 to a patient with normal pulse ox.  When you are at ABEM General get the highest-level test.  Just get an MRI if you need it.  ABEM General has every resource.


Paik        M&M  and Asthma Management


John presented 3 cases.  I am not describing the specific cases to maintain confidentiality.


Asthma is very prevalent  affecting greater than 17milion Americans.   There were 1144 reported  in-hospital deaths 2006-2008.



*Risk Factors for Death from Asthma


Management to Avoid Intubation

IV Magnesium

Bipap

Next management tool is Epi IM or Terbutaline Sub Q

Heliox can be used to improve air movement thru bronchioles.

Ketamine can be used for sedation and it also has bronchodilator effects

All the above maneuvers do not have good data supporting their use but all are inexpensive and pretty benign so all are still reasonable moves for severe asthma.


There was a discussion about how to manage a pH <7  secondary to respiratory acidosis in the intubated asthmatic.  Careful increases in minute ventilation with an eye on plateau pressures was the first management move of most attendings present.  Most would not use IV bicarb to alter pH.   THAM would be an option in this situation to manage the pH in addition to increasing the minute ventilation.   Bolus THAM 250ml then give another 250 ml as a drip over 1-2 hours. You have to be sure the patient is making urine so they can clear the THAM.  Hyperkalemia and hypoglycemia are side effects of THAM.



ACMC ED Handoff Protocol


Quiet Environment with limited interruptions

Both faculty and residents sign out together

We follow an SBAR-type format

A record of the handoff is entered in the electronic medical record

We do bed-side handoff for critical care patients

 

 

Carlson              Opioids

 

Opioids are semi-synthetic or synthetic drugs altered from the parent opiate.

Synthetic opioids like fentanyl and methadone and tramadol will not show up on standard drug screens.  They are too dissimilar  of a molecule from morphine for the screen to pick it up.


If a patient is altered, look for and remove their fentanyl patches.  They can be hard to see and forgotten by the patient or nursing home staff.  Andrea had a patient recently who was found to have 5 fentanyl patches.   Fentanyl patches have 10mg of fentanyl total in the patch!   When you remove a fentanyl patch, do not just throw it out.  Put it in a sharps container.


 


*Opiate Toxidrome :  CNS depression, miosis, bradypnea, decreased bowel sounds

 

*Non-cardiogenic pulmonary edema is more common in patients who receive naloxone.  Naloxone may increase respiratory drive but not airway tone and patients may develop edema.

 

46 persons are believed to die every day from prescription drug overdoses.  There are more deaths from prescription drug overdoses than heroin and cocaine overdoses combined.



*Opioid Schedules

 

 

* Mu1 receptors cause euphoria,  Mu2 receptors cause respiratory depression,

Kappa 2 receptors cause dysphoria

Andrea suggested observing heroin overdose patients for 4 hours in the ED.  If they choose to sign out AMA, be sure you carefully document decisional capacity.

 

McDowell      Ketamine for ETOH Withdrawl

 

Standard protocol is lorazepam first line incrementally increasing.  If lorazepam at or above 6-8 mg IVP is ineffective then next give phenobarb 10mg/kg over 30 minutes.  If the patient is still having severe withdrawl, start a lorazepam infusion 5 mg/hr and titrate up to 30mg/hr.   Suggested as next line treatment is ketamine 0.25mg/kg/hr.

 

 

Nejak        Altered Mental Status

 

 

*AEIOU  Tips mnemonic



Some stuff you could miss with AEIOU TIPS,  thyroid storm, myxedema coma, adrenal crisis, CO2 narcosis, TTP, CO toxicity, toxic alcohols, ASA overdose, non-accidental trauma, non-convulsant status epilepticus, neuroloptic malignant syndrome, serotonin syndrome. 


Dan Nejak’s recommendation is to use EMR technology to help improve your differential.   He has an altered mental status template built into his documentation files that he can pull up in a chart.


Harwood comment: Vital signs can help you a great deal.  Bradycardia and hypothermia suggest myxedema coma.  ABG’s are a great way to gain rapid critical info on altered mental status patients. 

Christine Kulstad comment:  Beware of cognitive bias in the undifferentiated patient.   Give yourself a cognitive stop and give it some concerted thought.



Holland       ENT Infections

 

Otitis externa: Treat with pain control and Floxin drops or cortisporin otic suspension.  A wick may be necessary in the ear canal to get the antibiotic into the canal.  Beware malignant otitis externa due to pseudomonas.  MOE is more common in diabetics and immunocompromised patients.  Treat with intravenous anti-pseudomonal penicillin and aminoglycoside.  Also pay close attention to glucose control.


HIV can present with a mono-like pharyngitis.


*Centor Criteria for strep throat


There was a discussion about the management of peritonsillar abscess.  The majority of faculty felt that routine CT was not indicated for all peritonsillar abscesses.   If the patient has palpable fluctuance or ultrasound-identified abscess then attempt to drain the abscess in the ED with a needle.  If you cannot identify a drainable abscess or are unsuccessful draining the abscess, the patient may have peritonsillar cellulitis.  Peritonsillar cellulitis can be treated with IV antibiotics and IV steroids.  Some patients who do not have stridor or drooling can be discharged home with close ENT follow-up.  Dr. Regan noted that we did recently see in the ED a patient with peri-tonsillar abscess that required emergent intubation in the ED.  The larynx was very edematous when visualized with glidescope.  So you have to be a little cautious with who you manage as an outpatient.


Ludwig’s angina has edema of upper neck and floor of mouth.  Patients may have displacement of tongue.  Late signs are drooling and stridor.  With proper antibiotics edema may take up to 1 week to resolve.



Conference Notes 11-18-2015

Garrett-Hauser/Richard JamesChaplain         Religion and Medicine

 

We discussed the challenges of discussing and respecting religion with patients and their families.  The background for this discussion was based on the article: Religion, Spirituality, and the Intensive Care Unit.  The Sound of Silence.   Balboni, et al. JAMA Intern Med. 2015

 

Exerpt of Article Review: the authors explore the re-

ligious and/or spiritual thematic content of goals-of-care con-

versations between health care professionals and surrogates

of critically ill patients. Although religion was important to

77.6% of the surrogates, only 16.1% of the conferences in-

cluded any reference to religion or spirituality. Furthermore,

when they did occur, these conversations were initiated by sur-

rogates 65.0% of the time. A health care professional raised

spiritual concepts (eg, spiritual histories) only 14 times (5.6%),

and only 2 of the conferences (0.8%) were attended by a chap-

lain. When surrogates raised spiritual concepts, health care pro-

fessionals’ most common response was to change the subject

to the medical realities at hand. Although empathic re-

sponses were the next most common response, health care pro-

fessionals, in general, “rarely directly addressed surrogate’s

spiritual or religious language.” Only 2 health care profession-

als responded by exploring the patient’s or surrogate’s spiri-

tuality. Notably, for conversations that included religious and/or

spiritual content, various themes were identified, with miracles

being one of several spiritual themes that intersected with medical care

 

the authors highlight, indicate the crucial need for greater integra-

tion of chaplaincy into ICU care

4,6

and for spiritual care education for health care professionals,

5

including how to integrate

a basic exploration of religious and/or spiritual values into

health care communication.

Our patients and families who face serious illness typi-

cally find themselves in spiritual isolation in the medical set-

ting; their medical caregivers do not hear the spiritual rever-

berations of illness on their well-being and medical decisions.

As with the lonely, falling tree, the reverberations are unde-

niably there. The question remains whether we who care for

dying persons and their families will learn how to be present and listen.

 

 

 

Every spiritual discussion with patients/families is contextual.  You have to make an effort to “know thy patient” and “know thy self”.    Understanding the patient and your self will help you dialogue about religion.   You should try to rise above specific religious boundaries and get to a level of general spirituality.  A key is to make your interaction with the patient an experience of benevolent intent toward them.

 

You can actually express benevolent intent in a pretty short period of time in the ED.  As a non-medical example, you can tell even when you interact with a cashier for a even just aminute whether or not they are acknowledging and respecting you as a person.

Recognize when patients and families use their religion in unproductive ways.

An example would be a patient refusing surgery for a gangrenous leg and saying God will heal this leg.  You may need to consult a chaplain to help with this type of situation.

 

A good starter question to discuss spirituality with patients/families would be,

“Is there a particular faith perspective that you use to cope with life’s challenges?”

 

Harwood comment:  If someone expresses that faith is an important part of his or her coping strategy, I would consult pastoral care to help with the case.   Chaplain Richard James agreed.   The key is to show respect for a person’s faith.

 

 

Bamman         Traumatic Hand and Wrist Injuries

 

 

*Brief Hand Neuro Exam.



*Ever Briefer Neuro Exam.  All 3 nerves in one movement.

 

 

 

*Fight Bite Injury.   This is a high risk injury.  Give IV antibiotics such as Unasyn or Clindamycin/Cipro if PCN allergic.   GetX-rays to evaluate for fracture and foreign body.  Examine the wound through the full range of motion to identify any tendon injury.  Consult Hand Surgery. 


We had a discussion about nail trephination.  Girzadas suggested doing a digital block or at least discussing options for pain control or making the patient aware of possible pain prior to the procedure.  All the other attendings strongly disagreed and felt no digital block was necessary.   They felt just warning the patient that they may feel a momentary pain was adequate.


Mark discussed finger tip injuries.  All attendings agreed that if the nail is fully intact, don’t remove the nail to go searching for a nail bed laceration.  Only remove the nail to repair the nail bed if the nail has already been partially avulsed or disrupted.



*Thumb Fractures   There are two syllables in Bennet and the Bennet’s Fracture has two fracture pieces.  Rolando has three syllables and has three or more fracture pieces.




*Lunate and Peri-lunate Dislocations.  These need Hand Consultation while the patient is in the ED.  



*Scapho-Lunate Dissociation



*Colle’s Fracture


*High Pressure Injection Injury is a surgical emergency.



Williamson        Study Guide     GI

 

CT is not indicated for uncomplicated pancreatitis. 

Pancreatitis is considered severe when there is pulmonary dysfunction.

The most common causes of pancreatitis in order from most to least common:   Gallstones, alcohol, idiopathic, hypertriglyceridemia, scorpion bite.



*Ranson Criteria


*Epiploic appendigitis  is due to torsion or inflammation of an epiploic appendage.



Most common cause of SBO is adhesions.


*Olgilvie Syndrome seen most commonly in elderly patients who are ill, have had trauma or surgery.   It usually resolves with NG drainage alone.


*We discussed TXA for Upper GI Bleeding.  A quick literature review found a  Meta-Analysis showing some evidence of  lower mortality in patients who received TXA. 


The most common cause of bacterial diarrhea in patients diagnosed in the ED is Campylobacter.  Faculty all agreed that probiotics or yogurt with active cultures is a useful management tool for patients with diarrhea.


Elise comment:   Pepto-Bismol can turn a patient’s tongue black and can turn their stool black.


Staph Aureus Gastroenteritis:  Mayonaise/potato salad is a common buzz word in questions,  onset occurs within a couple of hours, patients have mostly vomiting,  it is self-limited.  Resolves in a few hours.


*Crohn’s vs Ulcerative Colitis


Most common cause of cancer of biliary tree is cholangio (gall bladder) carcinoma. 

 

 

Alexander        EKG Workshop

Ari led the residents thru multiple clinical scenarios of tachycardia with EKG stimuli. 


Elise comment: Regular rhythms (both supraventricular and ventricular)are more easily converted than irregular rhythms so initially use 100J for these regular rhythms.   Irregular rhythms are more difficult to convert so use 200J inititally.


Pharmacist comment: Procainamide is favored over amiodarone for converting stable V-Tach.


Ari discussed a case of SVT in a patient  who was on digoxin and a beta blocker.  The patient was given adenosine and then developed an idioventricular rhythm for a short time before she converted to sinus.  It was pretty terrifying for a few minutes. This may be a risk in patients on other cardiac meds.


Katiyar/Bonaguro   QI     Sepsis


Please pay close attention to the Sepsis Guidelines.    Document the suspected source of infection.    You need to perform/document a repeat physical exam after the patient receives their initial 30 ml fluid boluses.



*Surviving Sepsis Guidelines


*Sepsis Definitions

To document your repeat exam you need to note the following:

Vital signs, heart exam, lung exam, cap refill, peripheral pulse and skin exam.

In our Cerner system if you type in ..sepsis it will pull up a simple templated note that will draw in the vital signs and then you just need to fill in the physical exam.

Abhi and Sheila comments:  Think of sepsis similar to belly pain where you want to get a repeat exam documented in the chart.


The clock does not start toward time targets until there is documentation of  a suspected or confirmed infection in the chart or an elevated lactate is identified.


Conference Notes 11-11-2015

Nejak        Adrenal Crisis and Thyroid Storm

 

Adrenal Crisis is a life-threatening exacerbation of adrenal insufficiency.  The most common cause worldwide is TB.  The most common cause in the western hemisphere is autoimmune. HIV is another common cause.   The most common cause of secondary adrenal insufficiency is withdrawal of steroid therapy (both oral and inhaled steroids).  Also be aware of post-partum adrenal insufficiency (Sheehan’s syndrome) and post-head trauma adrenal insufficiency.

 

Adrenal crisis can present with hypotension, altered mental status, abdominal tenderness, hyponatremia/hyperkalemia, hypoglycemia, fever, and hyperpigmentation (hyperpigmentation only with primary adrenal insufficiency).   These are all signs that can be easily attributable to other diseases.  So it is a difficult diagnosis to make.

 

Treat adrenal crisis with D5.9NS and hydrocortisone 100mg IV bolus Q 6 hours for first 24 hours.  Use pressors as needed.

 

Thyroid Storm presents as hyperthermia, tachycardia, and altered mental status.

 

 

*Diagnostic Scoring for thyroid storm.  There was a discussion that this scoring system may have difficulty differentiating sepsis and thyroid storm.

 

 

*Treatment of thyroid storm.  Dan recommended: IV fluids, cooling, PTU followed Iodine, esmolol drip, steroids, and cholestyramine.  Cholestyramine interrupts enterohepatic circulation of thyroid hormone.

 

 

Parker          Pearls from Podcasts

 

Braden played an excerpt of “This American Life” podcast describing a haunting of a house.  This description was reported in a 1921 case report in the Journal of Ophthalmology.   The experiences  (strange sounds, visions, tactile sensations)the family had were due to CO toxicity in the house.

 

CO toxicity can present with headache, nausea, and a myriad of other nonspecific symptoms.   You have to have a high index of suspicion for CO toxicity especially in the cold weather months when people are using their furnaces. A CO level over 9% is indicative of CO poisoning. Use to the finger probe device to check the level.  It is accurate and painless.   Treatment is O2 and consider hyperbaric O2.    Andrea comment:  Headache is usually present with CO poisoning.  In the winter ask the patient if other people in the house are also having headaches. CO toxicity will affect everyone in the house.

 

SMARTEM Podcaston who needs LP to identify SubArachnoid Hemorrhage.

 

CT negative/LP positive SAH are aneurysmal and amenable to neurosurgical intervention.  The problem is identifying these patients.

 

*SubArachnoid Hemorrhage

 

Using CTA to screen for aneurysmal hemorrhage are problematic because 3% of the population have asymptomatic aneurysms that don’t require intervention. 

 

Doing an LP with every bad headache is also problematic.  It is estimated you will have to do  about 1500 LP’s to identify 1 patient who has a negative CT and a SAH due to aneurysm that is amenable to clipping.

 

Only 20% of SAH have sentinel bleeds.  If the sentinel bleed is missed the patient’s subsequent morbidity and mortality is 45%.  If the diagnosis is made at the tiem of the sentinel bleed,  the patient’s  morbidity and mortality is marginally better at 33%.

 

CT is very sensitive (99%) for picking up SAH within 6 hours of headache onset.   After 6 hours the sensitivity drops to 86% .  The other factor is that patients who present after 6 hours from the onset of headache have half the risk of SAH compared to patients who present within 6 hours of onset of headache.

 

Basically a negative CT within 6 hours of onset of headache effectively rules out SAH.   If the patient presents after 6 hours of onset of headache and the CT is negative, look for high risk features that would increase a patient’s risk and use shared decision making with the patient  to decide whether to do the LP.

 

Elise comment: Be careful.  This can be a tricky diagnosis.  If you are worried do an LP.   Harwood comment:  Be alert for the thunderclap headache.  If the patient reports that the headache became maximal within one hour after onset, that is considered a thunderclap headache.  There are a lot of urban legends about LP’s and patients in general will choose not to have an LP.  If you want to work the patient up further and they are refusing LP you can skip to CTA.  There is however a risk of finding asymptomatic aneurysms as noted earlier.   Girzadas comment: I recall a patient with a severe headache who had a negative CT and LP but still had a really bad headache. We next did a CTA and found an intracranial carotid dissection.  Be aware that this diagnosis is also lurking out there.  If the patient looks bad and the ED work up is negative use your tools to keep looking (neuro consult, CTA, MRA, etc.)

 

*Carotid dissection

 

Munoz     Pediatric Rashes

 

 

*Roseola   Fever usually starts after fever goes down.  Kid is usually happy.  Can be associated with febrile seizures.

 

* Pityriasis Rosea   Thought to be due to virus.  Can be mildly pruritic.  Has a larger herald patch.  Can last 2-3 months.

 

 

*Chicken Pox   Pruritic vesicles in various stages of evolution.

 

 

Snip20151111_8.png

*Measles

 

 

*Scarlet fever    Due to GABS. More common under 10 years old.  Sandpaper feel to the rash.

 

 

*Pastia Lines of Scarlet fever and Sandpaper texture

 

 

*Rocky Mountain Spotted Fever

 

 

*Erythema Chronicum Migrans Rash of Lyme Disease

 

 

Urumov/Navarette     Oral Boards

 

Case 1.  39 yo male with history of HIV presents with altered mental status. Exam shows evidence of Kaposi’s sarcoma on the skin.  LP studies are positive for india ink and cryptococcal antigen.  Treatment is active rewarming, IV anti-fungal therapy (Amphoterecin B and flucytosine).  In immunocompromised patients always do a CT prior to LP to identify an potential mass lesions.  The cryptococcal antigen study is more sensitive than an india ink test.  You need to do that test on all HIV patients with altered mental status who you tap.

Elise comment: You have to know to do CT/LP and crypto antigen studies in HIV patients who have headache or have altered mental status.  Prior to LP, do CT with and without contrast to look for ring enhancing lesions.  If the patient has Cryptococcus and their opening pressure is high you need to remove fluid to lower ICP.  Harwood comment: You can fill your LP tubes to take off fluid.   No one knew the exact amount of fluid to take off but most felt thatif you took off 20-40 ml that should help a lot.

“CSF pressures should be reduced by therapeutic CSF removal when the opening pressure exceeds 250 mm H2 O. Following removal of CSF, the closing pressure should be less than 200 mm H2 O or at least 50% of the elevated opening pressure.  Medscape Reference”

 

Case 2.  68 yo female with headache and vomiting.  Patient became ill whe she attended a movie with her husband.   On exam patient is found to have one eye with a poorly reactive pupil and elevated intraocular pressure.  The diagnosis is acute angle closure glaucoma.

 

*Angle closure glaucoma

 

*Angle closure glaucoma treatment

 

Case3.   14 month old child who abruptly started coughing while playing.  No fever or URI symptoms.   CXR shows coin in esophagus.

 

*Coin in the esophagus (at the clavicles, george washington facing forward).   To be honest, I could not find a picture of a coin in the trachea (coin sideways, overlaying trachea).  It must be pretty rare.  The coin size must not fit thru the chords readily.  Tracheal foreign bodies probably have to be generally smaller than a coin.

ENT usually removes a FB lodged in upper esophagus and airway.  GI usually removes FB’s lodged in the lower esophagus.

 

 

Kristen Dibenedetto     Labeling Specimens

 

Bring the lpatient labels into the patient room prior to obtaining the sample or doing the procedure. Verify that the label matches the patient.  Ask the patient their name, then look at their wrist band.  Check that their name, birthdate, and medical record number all match the labels you have in your hand.  

Be sure you put your initials, date, and time on the label at the bedside. 

 

Lastly when you put the specimens in the specimen bag at the bedside say out loud the last three numbers of the MR number and match them to the patient’s wrist band at the bedside.

 

We wrapped up with a robust discussion about the difficulties with patient labels in the ED.

 

 

Tekwani       Study Guide   GI

 

Acute cholangitis: Fever/Pain/Jaundice=Charcot’s Triad.   Add in hypotension and altered mental status and that=Reynold’s Pentad.    Most common causative organisms are ecoli and other gram negatives.

 

Spontaneous bacterial peritonitis is identified by ascites fluid showing WBC>1000 or polys >250.  Enterobacter is the most common causative organism.   Treat with Rocephin or Zosyn.

 

 

*Acetaminophen metabolism pathway.  Acetaminophen overdose is the most common cause of liver failure in the US.

 

*Amebic Liver Abscess

Amebic liver abscess is caused by entamoeba histolytica.  E histolytica exists in 2 forms. The cyst stage is the infective form, and the trophozoite stage causes invasive disease. People who chronically carry E histolytica shed cysts in their feces; these cysts are transmitted primarily by food and water contamination. Rare cases of transmission via oral and anal sex or direct colonic inoculation through colonic irrigation devices have occurred. Cysts are resistant to gastric acid, but the wall is broken down by trypsin in the small intestine. Trophozoites are released and colonize the cecum. To initiate symptomatic infection, E histolytica trophozoites present in the lumen must adhere to the underlying mucosa and penetrate the mucosal layer.

Liver involvement occurs following invasion of E histolytica into mesenteric venules. Amebae then enter the portal circulation and travel to the liver where they typically form large abscesses.

The right lobe of the liver is more commonly affected than the left lobe. This has been attributed to the fact that the right lobe portal laminar blood flow is supplied predominantly by the superior mesenteric vein, whereas the left lobe portal blood flow is supplied by the splenic vein.   Medscape Reference.

 

 

 

*Hepatitis B Serology:  HBS AG is on the surface of the virus.   HBS AB is the antibody to HBS AG.    HBC AG is in the core of the virus.  HBCAB is the antibody to the core.  HBE AG is the degredation of the core and only seen in acute infection.   Anti-HBC IGM is also only seen in acute infection.


Irritable Bowel Syndrome pain is relieved by defecation.


Large bowel obstruction is most commonly caused by malignancy.


Most common cause of massive lower GI bleeding is diverticulosis.


For the diagnosis of pancreatitis, amylase and lipase have similar sensitivities but lipase is more specific.  So most clinicians order a lipase and pass on the amylase.


Toxic megacolon is more common in ulcerative colitis.  Ulcerative colitis almost always involves the rectum.  Crohn’s disease more commonly has peri-anal involvement.


Hart     Safety Lecture


We discussed issues with safe disposal of sharps.


If you identify that a sharps bin is full and needs to be replaced, call EVS at 41-5958 and they will come and replace the bin.


Do not try to force sharps into a full sharps bin.  Call for a replacement bin or carry your sharps carefully to another bin.   We discussed the possibility of having a large sharps bin on wheels that we could move to where a procedure is performed in case there is not an easily available, not completely full sharps container.


Don’t recap needles.  If you do recap a needle, use a one-handed technique.


To more easily dispose of a guide wire, you can coil up a central line guide wire with the rubber band that you used for the ultrasound probe cover.   Christine comment: Alternatively you can use the plastic guidewire sheath to replace the wire.
















 

 

 

 


 

















Conference Notes 11-4-2015

Barounis     Septic Shock

 

Early intervention in the ED affects the trajectory of sepsis illness throughout the hospital course.

The mortality rate of septic shock is still @20%.  Death from sepsis is due to multi-organ dysfunction syndrome.  Early intervention can prevent the development of multi-organ dysfunction syndrome.

 

There has been no single clinical intervention that has been shown to improve mortality from sepsis.  However, sepsis mortality has decreased over the last decade.   This is likely due to earlier diagnosis and more attentive care of sepsis patients.

 

SIRS criteria are insensitive for identifying all sepsis patients. There is no definitive biomarker to identify sepsis.  Clinical evaluation and judgment are the best tools to identify sepsis.

 

Encephalopathy=acute brain failure.   It is a sign of sepsis.

Other signs indicate sepsis: Hypotension, tachycardia, oliguria, acute brain failure, poor peripheral perfusion, acidosis, and respiratory distress.

 

The management of hypotension is an emergency.

 

In all large RCT’s on sepsis the average volume of fluid that a septic patient received was 4 liters.  Most septic patients will likely need somewhere close to that volume over the first day or so.

 

Dave showed an echo in which the LV was hyper-contractile with the walls of the LV “kissing” in systole.  If the walls of the LV are touching in systole, the patient is likely hypovolemic or severely vasodilated.

 

You have to trend lactates every 4 hours in septic patients.

 

Chloride (normal saline) is a renal toxin and has been shown to increase the incidence of AKI in septic patients. Dave prefers LR because it is safe and more physiologic.  LR has potassium in it but it is still OK in patients with hyperkalemia.  

Dave made the point that in a recent study, healthy volunteers receiving 2 liters of NS had a transient decrease in their GFR.  Elise comment: I am not so worried about NS in a patient who needs 1-2 liters.  If a patient needs 3 or 4 or 5 liters you need to be giving LR. 

 

Dave’s first-line pressor is norepinephrine.   If a second pressor is needed he uses vasopressin.  If you have not corrected hypotension with these two pressors you need to take cognitive stop and say to yourself,  “What the heck is going on?” 

 

 

Motzny Study GuideEMS

 Unfortunately I missed the majority of this excellent lecture.

 

Bioweapons

 

Acute Radiation Sickness.   Most commonly noted indicator of poor prognosis is low total lymphocyte count on CBC.

Set up your cammand center for a HAZMAT response up-wind and up-hill.

The most common barrier to an effective disaster response is problems with communication.

 

Nejak   Oxygen Delivery

 

Dan did an awesome job discussing and demonstrating various oygen delivery devices available in the ED.

 

 

April Kennedy/Felder/Urumov     Dental Procedure Lab

 

Much Thanks to April Kennedy DDS for leading an outstanding dental procedure lab!

 

Snip20151109_1.png










Conference Notes 10-28-2015

Htet/Meyers   STEMI Conference

 

Case 1.  Patient with chest pain.

 

*Initial EKG

Cardiologist comments: It is unusual to have isolated V1 and V2 ST elevation without ST elevation in the other precordial leads.  If the ST elevation in V1 And V2 is related to proximal LAD occlusion it should also affect the other leads.  EKG also shows IRBB and RV strain.  It is also unusual for a STEMI to be bradycardic if it is not an inferior STEMI.

 

It was learned that patient had history of chronic aortic dissection and prior PE.  Patient was taken to the cath lab and a Type A aortic dissection was identified.

 

There was discussion of the risk of double dye load for CTA followed by coronary angiogram.  Dr. Silverman said that even if you take the patient to cath first without CT chest, the patient would still get about the same dye load for an aortic root shot followed by evaluation of the coronary arteries.   An alternative approach could be TEE for a patient with an elevated creatinine.

 

Dr. Silverman noted that pain from aortic dissection is more abrupt then that for AMI.

There was a discussion about the use of d-dimer to evaluate for aortic dissection.  Cardiology faculty was relatively  pro d-dimer for this indication.  EM faculty was more cautious about d-dimer in this setting.   Harwood and Elise felt d-dimer will miss about 5% of aortic dissections. Al-Kaled stated that if an elisa d-dimer is negative the patient does not have a dissection.  Drs. Harwood, C. Kulstad and Lovell almost had dissections of their own in response to this statement.  They felt the data shows that D-dimer can miss intramural hematomas/dissections.  Clearly this issue is controversial.

 

IRAD data on Aortic dissection shows that “classic” history and physical findings are not commonly present.

 

Elise comments and data: To be "low risk", you need a score of zero, meaning none of these features.  The patients I'm considering dissection in are usually patients who have abrupt, ripping, tearing pain, and therefore will not be "low risk".  Lower end of likelihood ratio 96%.  As Harwood said, not a disease to accept a 4% miss rate, and with the additional issue of low specificity, I'm not a fan of using d-dimer for diagnosing aortic dissection.

 

 

*Aortic Dissection Decision Rule

 

Cardiology comment: If patient has a pericardial effusion on echo in the setting of chest pain think aortic dissection.

 

Consensus by cardiology: Do a CTA if you consider aortic dissection.

 

Case 2.  Chest pain in a patient with recent stroke.

 

 

EKG

 

If you see a heart rate of 150 consider A-flutter.   If you see an RSR pattern in addition to a heart rate of 150 that makes A-flutter even more likely.  

CXR shows very prominent right pulmonary artery.  Bedside US showed dilated RV.  CTPE done and showed PE.

 

2.Massive vs. Sub-Massive PE

 

EKOS catheter was placed in pulmonary artery and TPA was given over 12 hours.

 

 

* EKOS catheter

 

*Ekos Catheter

Cindy Chan Follow Up Comment: Cards in PE management: The cardiologists have expressed willingness to assist with management of sub-massive and massive PEs, as many of them are credentialed to use the EKOS catheter.

- "On hours" 7a-7p: can call the on-call interventionalist for MASSIVE or SUB-MASSIVE PEs, as either they may be credentialed or should be able to get a partner who is help with this

- "OFF hours" 7p-7a: can call the on-call interventionalist for MASSIVE PEs to see if they are credentialed to assist; if not, then continue with our "traditional" course of management. SUB-MASSIVE PEs should be able to wait until the morning, when cards can be contacted if needed.

 

 

Iannitelli/Katiyar    Oral Boards

 

Unfortunately I missed this excellent lecture.

 

PharmD Lecture      Push Dose Pressors

 

Phenylepherine is the only drug at ACMC approved for bolus therapy.   It is a pure alpha agonist with rapid onset and short half life.

 

Avoid phenylephrine in patients who are bradycardic.  You could possibly cause worse reflex bradycardia.

 

ACMC Recommendation: 0.5-1ml (50-100mcg) of phenylephrine every 2-5 minutes.  Can be used in the peri-intubation period and peri-code period. 

 

Christine comment: Dopamine is universally available and is likely safer in an emergent situation.  You and your nurses are less likely to make a dosing mistake with dopamine than with phenylephrine.  The fail-safe way of giving emergent pressors is to just start a dopamine drip.

 

Mike Kennedy: Epinepherine could be used as a push dose pressor as well.  PharmD response is that there is no data on using epi as a push dose pressor.  It is not approved by our P&T Committee for push dose pressor use. 

 

Elise comment: I don’t use push dose pressors.  If I am resuscitating the patient in the peri-intubation period I will start a drip ofnorepi.  The only time I use epi in a push dose situation is for anaphylaxis.

 

There was a discussion of what the ultimate aim of push dose pressors.  You are just increasing BP transiently and not treating the underlying problem.  On the other hand, there is a growing chorus in the EM world about using push dose pressors in the peri-intubation period for very unstable patients ie. “Resuscitation before intubation”.  The patient oriented outcomes for this treatment will need to be studied further.

Urumov/Girzadas    Recruiting Season Update

A. Patel     Study Guide

Unfortunately I missed this excellent presentation.

 

 









Conference Notes 10-21-2015

Remke/E. Kulstad     Oral Boards

Case 1.  30yo male fell into the sand and surf while body surfing.  Patient has neck pain and paralysis of extremities.  Patient has decreased rectal tone and priapism.   C-spine x-ray shows compression fracture of C5 with bilateral facet dislocation.  Because patient had a C5 fracture and shallow respirations, he was intubated.   The patient developed hypotension and bradycardia due to neurogenic shock. Neurogenic shock  was treated with IV fluids and pressors.   Steroids are no longer indicated for spinal cord injury.  The patient was kept in C-spine immobilization.  Neurosurgery was consulted.

 

C5 Fracture and likely bilateral facet dislocation

 

Case 2. 12 yo male with left hand pain after having a stingray barb impaled into the dorsum of the left hand.   X-rays show no bony involvement.   Hot water therapy was initiated which greatly relieved pain.  The toxin of a stingray barb is exquisitely heat sensitive.  Hot water will significantly diminish pain.  The barb was then removed with trimming the end off and traction.

 

*Stingray Barb

 

Case 3. 22yo female passed out on the beach.  Temp=41.5C,  HR=136.    The patient had been playing volleyball and running on the beach in hot weather.  On exam, patient was confused and lethargic.  Skin is warm and dry.  (Three keys to heat stroke diagnosis: Temp >41, dry skin, mental status changes) Rapid cooling was initiated. Goal of cooling is 39C.  CPK was 9000 indicating rhabdomyolysis.  Aggressive hydration was initiated to treat rhabdomyolysis.   Avoid antipyretics in heat stroke because both liver and kidney are considered injured and may be very sensitive to acetaminophen and ibuprofen. 

Elise comments: A pulmonary function assessment (NIF) can be useful to assess respiratory strength in a c-spine injured patients.  

Look for any source of hemorrhagic shock in a trauma patient before you attribute hypotension to neurogenic shock. 

Treat slimy beach envenomation’s (jelly fish) with vinegar.   Treat sharp/pointy envenomation’s (stingray barbs) with hot water.   It’s actually not that simple but for test taking purposes it is pretty good rule of thumb.

 

Chan    Study Guide          Genitourinary Emergencies

 

2015 CDC Chlamydia Treatment Guidelines

 

2015 CDC Gonorrhea Treatment Guidelines

 

Before placing a foley in a patient with suspected urethral trauma, perform a retrograde urethrogram if you see blood at the meatus, a scrotal hematoma, or high riding prostate.   If you see none of these signs, gently try to place a foley.   If a foley is placed successfully, it acts as a stent  for the injury and should remain in place.

 

Pregnant women with asymptomatic bacteruria should be treated.  Asymptomatic bacteruria increases a pregnant woman’s risk of pyelonephritis.  Pregnant women with bacterial vaginosis either symptomatic or asymptomatic should be treated.

 

Testicular torsion has 96% salvagability in the first 6 hours.   

Detorsion attempts should turn the affected teste medial to lateral (opening the book).  Get the patient to surgery within 6 hours from onset of pain.

 

Opening the Book.&nbsp; Medial to lateral detorsion attempt

Opening the Book.  Medial to lateral detorsion attempt


*Ultrasound of torsed teste on right.  Left teste has normal flow.

 

90% of kidney stones smaller than 5mm will pass within 4 weeks.

 

5. *Kidney stones most commonly get lodged at these 3 locations: Ureteropelvic junction,  mid-ureter at pelvic brim,  and at the ureterovesicular junction.

 

Elise comment:  There are some downsides to Flomax in the elderly.  It can cause postural hypotension.   The newest interpretation of the data is that Flomax is most helpful in larger, upper tract stones.  Flomax is less efficacious for small stones far down in the ureter. 

 

Andrej Reference:

Quote from EM:RAP summary: December 2014:
Vincendeau, S et al. Tamsulosin hydrochloride vs placebo for management of distal ureteral stones: a multicentric, randomized, double-blind trial. Arch Intern Med. 2010 Dec 13;170(22):2021-7. PMID: 21149761.

This is probably the best study available: multicenter placebo controlled, randomized and double-blinded. They found no difference in expulsion rate (77% versus 71%) and no difference in the days to expulsion. They concluded that there was no benefit for small distal stones less than 7mm.
 

 

 

 

6. *Phimosis vs Paraphimosis    Para=Around,   so a paraphimosis means the foreskin is retracted around the penile shaft.  Paraphimosis needs to be reduced emergently.  Options include: manual compression and reduction, osmotic agents, traction with forceps, puncture techniques, and dorsal slit procedure.

 

 

Parker     M&M

 

Case left out to protect confidential information.  Take home lessons will be discussed.

 

If a central line is in a questionable location on CXR, you can use the “bubble test” to verify that the line is in a vessel that communicates with the RA.   To do this you rapidly inject saline while visualizing the RV on ultrasound.  If the line communicates with the RA, you will see turbulence in the RA with rapid injection of saline into the line. 

 

7.  *US of RA showing turbulence in the RA with rapid injection of 10 ml of saline.

 

You can also use ultrasound to visualize the guide wire in the IJ, RA, or RV prior to dilating your access tract.  To improve your visualization of the wire, use your finger to compress the vessel near the US probe.  You should be able to see the wire move on US. 

 

Elise comment: In all lines no matter how well they went, for patient safety, I recommend visualizing the wire in the IJ with ultrasound prior to dilating the vessel. 

 

Braden also discussed a technique of placing the angiocath in the central line kit over the wire without dilating the tract.  Then hooking up plain IV tubing or the wire holder in the central line kit to the angio cath to see if blood tracks up the tubing more than a few centimeters.  If it does you likely have entered an artery. 

 

You can use an ABG to detect the location of your line.  You need to draw an ABG from both the line and the radial artery and compare them.  This method can be problematic if you inconclusive results or you get some venous blood on the radial stick.

 

If you do inadvertently place a catheter in the carotid artery, don’t pull it out.  Call vascular for their consultation.  Pulling the line without surgical control can cause large hematomas, CNS events, and airway issues.

 

If you puncture the carotid artery with the needle but don’t dilate or place a catheter in the vessel, you can just pull out the needle and hold pressure for 15 minutes.  Needle puncture of the carotid is less problematic than dilating and canulating the vessel.

 

Ophthalmology Lab

Thank you to the Chiefs and Faculty for this outstanding workshop!

 

 

 

 

 

 

 

 

 

 

 

Conference Notes 10-14-2015

Special thanks to guest scribe Dr. Elise Lovell!

Conference Notes 10.14.2015

8 am:  Oral Boards:  Harwood vs. Paik

Case 1:  20 month female with abdominal pain/vomiting.   Lead poisoning:  Look for neuro sx, GI sx, hematology.  May show up as radio-opaque flakes on KUB.  CBC with basophilic stippling.  Lead levels useful.  Need to notify Health Department, call Tox Consult, remove from source, assess other kids.  Treatment Dimercaprol (BAL) IM, followed by Ca Na2EDTA.

Case 2:  20 yo male unstable Transmediastinal GSW. Need Echo, CT Chest, potentially bronch/esophagus.  Codeà ED thoracotomy; Shock/>1500 Chest Tube output immediately or 200 cc/hour for 3 hoursà OR; Stableà workup.

Case 3:  30 day female, ALTE:  Pertussis.  Reason for apneas in neonate:  bronchiolitis (RSV or influenza), pertussis.  May have 6 weeks of cough.  Treat with macrolide, exposures to be treated as well.   Diagnosis with PCR.  Admit if apnea, seizure, pneumonia, age <4months, respiratory distress, poor feeding.  Immunize!

 

9 am:  Radiology-Head CT West and Ede:

Use systematic approach:  “Blood Can Be Very Bad” = blood, cisterns, brain, ventricles, bone

Bleeds:

Epidural hematoma, lentiform, does not cross suture lines.  Usually arterial (middle meningeal classic).  May have lucid interval then decompensate.

Subdural hematoma, crescent shape, does cross suture lines, usually venous, may be indolent, with non focal neuro exam.

Subarachnoid hemorrhage; Hunt and Hess classification, 75-80% aneursymal, CT excellent sensitivity within first 6 hours then drops off.

Intracranial hemorrhage, hypertensive -> basal ganglia.  

Ischemic CVA:  often negative CT in first hours of presentation.  Look for hypoattenuation/sulcal effacement, blurring of basal ganglia in MCA infarcation, dense MCA sign (clot in MCA), insular ribbon sign-hypodensity and swelling of insular cortex.

Infections:  in meningitis CT before LP if AMS, focal neuro deficit, seizure, concern for mass (malignancy, papilledema).   Viral encephalitis CT usually normal initially, think about temporal changes for herpes.

Herniation:  6 types.  

9:30 am:  Syncope Lee:

Is it really syncope, or is it seizure , vertigo, etc?

Is it isolated primary syncope or is there an underlying cause (secondary syncope-cardiovascular, neurology, medication related)?  Be more worried if no prodrome symptoms (drop attack), or if exertional syncope.

History and physical, including PMH,  are key to identifying the cause.

In the young, think about dysrhythmia, seizure, pregnancy, blood sugar/electrolyte/volume status, bleeding/anemia.

Hypertrophic cardiomyopathy (LVH + Q waves)

Brugada (incomplete RBBB + downsloping ST segment)

WPW (delta wave + short PR)

wpw ekg.jpg

Arrhythmogenic Right Ventricular Dysplasia (epsilon wave-upright wave, looks like an Osborn wave in a normothermic patient)

Who can go home?  Low risk factors:  primary syncope without concerning historical factors, age <50, no h/o CV disease, normal exam, normal ECG

San Francisco Syncope rule:  H/O CHF, Hct<30%, abnormal ECG, SOB, SBP < 90.  Careful-initial sensitivity of high 90s hasn’t held up well in other studies, but need to apply in correct population...a patient with a SAH would be low risk by this rule.

Rose syncope rule adds BNP,  low O2 sat, in order to increase sensitivity.

None of the many rules are terrific....

CT extremely low yield-don’t do it unless high suspicion for CNS etiology, or if on anticoagulants, or if unwitnessed and old.

Can’t miss diagnoses?  PE, SAH, Aortic dissection, AAA, ectopic pregnancy.

 

10:00 am:  Toxicology Carlson:  Aspirin toxicity

Shows up in variety of products (arthritis meds, IBD meds)-beware of combination products and brand names such as BenGay, Peptobismol, Alkaseltzer!

Enteric preps unpredictable absorption.

Low Volume of Distribution (good for dialysis) and high protein binding (bad for dialysis but can affect this by changing pH). 

Pathophysiology/Clinical presentation:  Key ABG:  respiratory alkalosis + metabolic (lactic) acidosis.  Will see hyperventilationàstimulates resp. center in medulla.  Also stimulates vomiting center, causing GI loss of potassium.  Uncouples oxidative phosphylationà metabolic acidosis and fever.  Hypo or hyperglycemia.  Relative CNS glucopeniaàadds to AMS.  Direct ototoxinà tinnitus.

Single OD: 

Mild: Tinnitus, N/V, hyperventilation hyperpyrexia

Intermediate:  Agitation, dehydration, acid/base problems, non-cardiogenic CHF

Before they die:  Lethargy/coma, hyperthermia, cerebral edema, seizure, oliguria, hypotension.  A death from salicylate poisoning is a CNS death-watch their mental status!

Much less predictable symptoms with chronic OD, associated with delay in diagnosis, have higher CNS burder, more common in elderly, greater morbidity at lower levels.

Intubation-Keep Respiratory Rate High!!  Avoid iatrogenic acidosis.

Treatment: 

Charcoal-absorbs aspirin very well.  Skip it if > one hour or if already symptomatic.  Multiple dose no longer recommended.

IV Bicarb/urine alkalinization for ion trapping (bound vs unbound salicylate).  Need to be sure to supplement potassium!!  Body will resorb potassium if low, and waste HCl, so urine will stay acidotic unless potassium replaced.  Follow urine pH, not ABG. Goal of urine pH > 7.5.

Dialysis!!  If AMS, ARDS/CHF, failing standard therapy, pH , 7.20, level >90 mg/dl.  In chronic, symptoms trump level.

 

11:00 am:  ENT/Epistaxis:  Alexander

Assess if blood thinner use/predispositions for severe bleeding.  Consider CBC, coags.  Anterior= more common.  Posterior=more serious. 

Anterior:  Kiesselbach’s plexus, often secondary to mucosal dryness, conservative measures to manage

Posterior:  look at back of throat, often elderly patients with inherited or acquired coagulopathy.  More challenging to control.

Treatment:  ENT box!  Get afrin, viscous lidocaine.  Ask patient to blow nose, expel clots, in order to help vasoconstrictors to work.  Direct pressure with fingers or 2 tongue depressors taped together for 10-15 minutes.  Good lighting-use nasal speculum.  If no anterior source visualized, consider posterior bleed.  Use Frazier catheter as needed.  Consider silver nitrate cautery if you see specific source of bleeding, cauterize around the bleeding site.  Be prudent-can cause septal perforation.  Thrombogenic gels and foams can help.  Tranexamic acid (antifibrinolytic) can be used as topical application of injectable form. 

Packing:  Rapid Rhino balloon-soak balloon in water for 30 seconds before insertion.  Blow up with air.  Insert parallel to floor of nose.  Alternative preformed sponge-Merocel.  Consider coating with bacitracin ointment.  Wet with saline AFTER insertion.  For posterior pack, consider parenteral analgesics.  Inflate posterior balloon first, then anterior balloon.  Use minimal inflation necessary to control bleeding-can cause nasal necrosis!

Ant/Post balloon packing:

Antibiotics (Keflex) to help prevent sinusitis/toxic shock syndrome.

Admit if posterior packing required (airway obstruction, apnea, hypoxia, syncope/cardiac dysrhythmia possible).

Failed management-call ENT, may need embolization.  Also “greater palatine foramen block” with 2 cc of lido + epi.  Dr. Sherman hasn’t done it, so beware...

 

11:30 am:  Pulmonary SG:  Lovell

Primary spontaneous pneumothorax-if stable, consider treating with aspiration +/- Heimlich valve catheter, regardless of size of PTX.  They usually don't need chest tubes!

Tuberculosis

•Primary infection (usually asymptomatic, infrequently pneumonitis)

•Latent (+PPD, Ghon complex on CXR reflecting healed primary)

•Immunocompetentà5% risk of progressing to active disease within 2 years, 10% lifetime risk

•Reactivation-fever, night sweats, hemoptysis, weight loss

HIV-most common cause of pneumonia is Streptococcus pneumoniae.  Pneumonia in HIV associated with high rate of bacteremia.  Pleural effusions are common.  Think about TB with CD4 counts 250-500 (not just seen in low CD4 counts).  If pneumothorax, think PCP.

High risk factors for lung CA:  male, smoking, age > 40, no lower resp. infection symptoms.  If abnormal CXR or high risk factors or active bleeding, needs CT + bronch.

Pleural effusions:  transudate vs exudate

Exudate if:

•Pleural fluid /serum protein > 0.5 or

•Pleuralfluid /serum LDH > 0.6 or

•Pleural LDH > 2/3 upper limit serum LDH

Asthma and pregnancy:  Beware of CO2 retention!  During exacerbation, normal alkalosis of pregnancy is aggravated, leading to decreased placental blood flow.  Hypoxemia is usually more severe in fetus than in mother.

•PaO2 < 70 = severe hypoxemia

•PaCo2 > 35 = respiratory failure

•B2 agonists, inhaled and oral steroids all safe

Bronchitis

•GOLD:  Chronic:  treat COPD exacerbations with antibiotics if increased SOB with increased sputum volume/sputum purulence, or intubated

• ACCP:  For acute bronchitis, the routine use of antibiotics is not justified

Lung Transplant Pearls:

•Usually receiving tacrolimus (over cyclosporin) and mycophenolate mofetil (over azathioprine) and prednisone.

•Lots of drug toxicity and drug interactions-renal insufficiency (tacro and cyclo)

•Fever, cough, sputum, CP, FEV1 decline = rejection or infection.  Isolate.  Bronch.

•Most frequent cause of death long termà bronchiolitis obliterans (wheezing, tx antirejection agents)

Conference Notes 10-7-2015

Jeziorkowski        Eye Emergencies

 

 

Snip20151008_1.png

Stye vs Chalazion

 

 

Indications for Ophtho Repair of lacerations: Involvement of lid margin,  laceration anywhere near the medial canthus, laceration of inner surface of the lid, Ptosis, horizontal laceration with fat exposed

 

 

*Dacryocystitis Dacryocystitis is treated with oral augmentin and topical erythromycin.

 

 

Neonatal conjunctivitis within 2-5 days of birth consider gonorrhea.    Within 5-14 days of birth consider Chlamydia.   Both infections require oral and topical antibiotics. 

 

 

*HSV Keratitis   This is a potentially vision-loss infection.   Consult with Ophtho while the patient is in the ED. 

 

 

herpZosterOph.jpg

*Herpes Zoster Keratitis.   Look for Hutchinson’s sign, which is a zoster lesion on the tip of the nose.  Huthinson’s sign is a marker of corneal involvement by the herpes zoster virus.

 

 

When evaluating kids with eyelid swelling it is usually not orbital cellulitis.  Bug bites, pre-septal cellulitis, and allergic reactions are much more common.   If the child has normal eye movement, normal pupillary function, may have a sensation of pruritus, appears non-toxic with no fever, treat it as pre-septal cellulitis.  Give the patient oral antibiotics and discharge home.   If you have concern for orbital cellulitis: the child has fever, sicker looking kid, the child has eye pain, no pruritus, limited eye movement, start IV antibiotics and don’t get a CT on day 1.  Obtain a CT only if the patient is not improving on IV antibiotics.  Early CT  will rarely will change management.

 

 

Subconjunctival hemorrhages are generally benign.  Ask the patient  about coughing, sneezing, anticoagulant use.   Tekwani comment:  If you have a 360 degree subconjunctival hemorrhage you may want to evaluate for globe rupture.  Consider a CT of the orbit if there is a history of trauma.

 

*Retrobulbar Hematoma with vision loss and non-reactive pupil should be treated with lateral canthotomy.


*Hyphema   Keep head elevated.  Avoid NSAID's.  Consult Ophtho.

 

*Central Retinal Artery Occlusion.  Treat with eye massage, anterior chamber paracentesis by Ophthalmology, having patient breathe in a paper bag, and possibly hyperbaric therapy.

*Acute angle closure glaucoma.  Treat with mannitol, alpha agonist, beta-blocker, pilocarpine, and a carbonic anhydrous inhibitor. Consult Ophthalmology while patient is in ED

 

Navarrete     Review of DKA

 

Typical presentation is polyuria, polydipsia, and polyphagia.   Patients commonly will have nausea, vomiting, abdominal pain and body aches.

 

DKA is a problem of too little insulin.  Hyperglycemia is the symptom, not the problem.   HHS and DKA are on a spectrum.  HHS patients have a little bit of insulin to limit ketosis but not hyperglycemia.  HHS is less common and has higher mortality.

 

Cerebral edema occurs in <1% of DKA cases.  Cerebral edema is more common in kids and patients who present with more severe acidosis.  Avoid over-aggressive fluids.  Avoid insulin bolus and bicarb drips.   No one is sure of the cause.  Attending consensus was to be cautious with IV fluids in kids.   If you think the patient has cerebral edema, treat with hypertonic saline5 ml/kg over 10 minutes. 

 

When giving fluids follow the DKA protocol.  You need to be measured with your fluid resuscitation.  Adults can handle an initial first liter of saline then start your hourly rate.  Kids at most should receive an initial 10ml/kg bolus.  If the pediatric DKA patient is not in shock don’t give a bolus.   Just start an hourly rate per protocol. 

 

Don’t bolus insulin.  Start a drip at 0.1u/kg.   Follow the protocol from there. Bolus insulin increases the risk of cerebral edema.

 

VBG’s are adequate for DKA testing.  pH, bicarb and lactate are very consistent between VBG and ABG.

 

ETOH intoxication blocks hepatic metabolism of  glucagon and can result in hypoglycemia in the diabetic.   For the same reason,  glucagon won’t work to correct hypoglycemia in the intoxicated patient.

 

Lambert    Ultrasound ofthe Abdomen

 

 

*Gallstones with shadowing

 

*WES (wall echo shadow) Sign

 

Measure the gallbladder wall thickness at the anterior wall of the gallbladder.  The upper limit of normal gallbladder wall thickness is 3-4mm

The upper limit of normal for gallbladder size is 4cm in the AP diameter and 10 cm in the longitudinal diameter.

 

 

Snip20151008_8.png

*Hydronephrosis Grades 1-4

 

When scanning the abdominal aorta start in the epigastrium.  You will need to hold steady pressure with the probe to displace bowel.  Aim the probe slightly inferiorly to image in the plane of the aorta.  Keep the indicator on the probe pointed to the patient’s right side.   You have to scan the proximal, middle and distal aorta.  Most aneurysms are infra-renal.    Mike suggested also scanning the iliac arteries in older patients (over age 80) to look for iliac artery aneuryms. 

 

 

*AAA.  


Lambert and Team Ultrasound     Ultrasound Lab









 

 

 

 

Conference Notes 9-16-2015

Williamson/Parker     Oral Boards

 

Case 1.  50 yo male with respiratory distress.  HR=100 BP=140/80RR=34T=98.7   Patient became SOB at subway station.   Exam reveals copious oral secretions. Patient also vomited and had abdominal cramping. EMS notes that other persons in the subway have similar symptoms.

 

*Cholinergic Toxidrome

Diagnosis is cholinergic toxidrome due to Sarin (organophosphate gas).   Treatment is high dose atropine until respiratory secretions have improved significantly, intubation,  and 2PAM.  Consult poison control.   Patient needs to be decontaminated. Staff needs to use PPI to prevent secondary exposure to toxin.  Patient’s clothing can off–let toxin for 30 minutes.  Sarin is colorless and odorless.

 

Elise comment: I saw a patient die of organophosphate poisoning in Africa.  It was horrible. The patient basically drowned in his own secretions because we did not have enough atropine.    Andrea comment: Initial dosing of atropine is 2-6 mg and go up from there until secretions are dried up.   There are stockpiles of atropine around the city and country.   The poison center has access to these stockpiles.  You have to get 2PAM into the patient as soon as possible to prevent irreversible aging of the acetylcholinesterase.

 

Case 2. 16 yo male with a headache for the last 10 days.  Vitals are normal.   Patient notes a weird feeling and paresthesias from waist down.  Legs have spasms occasionally.  Bedside ultrasound shows urinary retention.  MRI of the spine shows transverse myelitis. 

 

Transverse_myelitis_MRI.jpg

*transverse myelitis

 

Diagnosis is transverse myelitis due to multiple sclerosis.  Treatment is IV steroids.  Patient required foley catheter drainage of urine.   Transverse myelitis is an inflammation of the spinal cord.

 

In most cases a sensory level is documented, most commonly in the mid-thoracic region in adults or the cervical region in children. Pain in the back, extremities, or abdomen is also common while paresthesias (e.g., tingling, numbness, burning sensations) are typical in adults. Sexual dysfunction is also the result of sensory and autonomic involvement. Increased urinary urgency, bowel or bladder incontinence, difficulty or inability to void, and incomplete evacuation of bowel or constipation are other characteristic autonomic symptoms. Spasticity and fatigue are other symptoms common to transverse myelitis. Additionally, depression is often documented in TM patients and must be treated to prevent devastating consequences.  (Transverse Myelitis Association)

 

Case 3.  47 yo male presents with right foot pain for one week.  Vitals normal except for heart rate of 102.   Patient states that the pain began after a heavy board fell on his foot.   X-rays show lis franc injury.

 

*Lis Franc injury

Treatment requires ORIF and non-weightbearing for 6 weeks.   Harwood comment: this is a pretty severe lis franc injury.  Lis franc injuries usually involve the first and second metatarsals.  This xray shows all the metatarsals dislocated. 


 

Levato     Community Acquired Cellulitis


For uncomplicated cellulitis with no abscess use either cefazolin or clindamycin.   Elise raised concerns about the level of clindamycin resistance at our institution.   Harwood added that he would prefer to give Bactrim if you suspect MRSA.  

Levato’s Bottom line:  use more ancef and less vanco.  Patients without comorbiditities who have straightforward cellulitis with no fluctuance have a strep cellulitis. 


*Algorithm for soft tissue infections (Thanks Elise)


 

Remke      M & M


In order to respect the confidentiality of M&M, I am not going to give case details of M &M’s.  I will just present the take home points.


Be sure you have ordered a type and screen on pregnant patients who are bleeding.  If they deteriorate  and need transfusion, waiting for the type and screen can cause delay or necessitate the use of O negative blood.


Quantify vaginal bleeding based on whether it is more or less than a normal period.

On pelvic exam, if there is clot or tissue in the os, remove it.  Removing tissue and clot from the os can help the uterus to contract and lessen bleeding.


*Liklihood of Ectopic Pregnancy

 

In the setting of pregnancy, consult OB service for heavy bleeding, open cervical os, and positive beta hcg with no iup seen on ultrasound. 

 

Things to tell patients:

25% of pregnancies have bleeding

If a viable fetus is seen on ultrasound the risk of miscarriage is less than 10%.

Miscarriage is more likely with heavy bleeding, bleeding lasting a week or more, and if pain is associated with bleeding.

 

Emergency Physician Cognitive Issues:

Pay attention to subtle vital sign changes

Double check that the labs you need are ordered like type and screen.

Listen to your nurses when they raise a concern.

Get help from consultants when you are faced with a difficult situation.

 

Kelly Williamson comment: RH negative Patients who have a subchorionic hemorrhage on ultrasound but are not bleeding into the vagina do not need rhogam

 

Ortiz-Romero     Imaging the Pediatric Acute Abdomen

 

On ultrasound, a normal appendix is smaller than 6mm, compressible and has no surrounding free fluid.  Appendicitis is larger than 6mm, non compressible and hyper-vascular.  There will be point tenderness over the appendix with appendicitis.  Unfortunately 70% of the time the appendix is not visualized with ultrasound.

 

If you have an Alvarado score less than 5 and have an ultrasound with no inflammatory changes without visualization of the appendix, that clinical scenario has a negative predictive value of 99% for appendicitis!

 

Value of Focused Appendicitis Ultrasound and Alvarado Score in Predicting Appendicitis in Children: Can We Reduce the Use of CT?

Blitman NM1, Anwar M, Brady KB, Taragin BH, Freeman K.

Author information

  • 11 Department of Radiology, Jacobi Medical Center, Albert Einstein College of Medicine, 1400 Pelham Pkwy S, Bronx, NY 10461.

Abstract

OBJECTIVE:

The purpose of this study was to evaluate the effectiveness of focused appendicitis ultrasound combined with Alvarado score to accurately identify appendicitis in children in whom it is suspected, thereby reducing unnecessary CT examinations and associated radiation exposure.

MATERIALS AND METHODS:

We retrospectively evaluated the focused appendicitis ultrasound, CT, clinical, and laboratory findings of 522 consecutively registered children (231 boys, 291 girls; mean age, 13.04 [SD, 5.02] years; range, 0.74 months-21 years) who underwent focused appendicitis ultrasound for abdominal pain in a pediatric emergency department from January 2008 through October 2009. All children underwent surgery or clinical follow-up to exclude missed appendicitis. Sonographic findings were characterized as positive, negative, or inconclusive (appendix not visualized). Alternative diagnoses were noted. Alvarado score (0-10 points based on multiple clinical criteria) was determined. Focused appendicitis ultrasound and Alvarado score results were compared with surgical and pathologic reports.

RESULTS:

Both focused appendicitis ultrasound results and Alvarado score were associated with likelihood of surgery for appendicitis (p = 0.0001). Focused appendicitis ultrasound had conclusive results: 105 positive and 27 negative in 132 of 522 (25.2%) children. In the 390 of 522 (74.7%) children with inconclusive focused appendicitis ultrasound findings, 43 of 390 (11.0%) eventually had a diagnosis of appendicitis with CT (n = 26) or Alvarado score (n = 17). Among children with inconclusive focused appendicitis ultrasound findings and an Alvarado score less than 5 (241/522, 46.1%), only one patient had appendicitis. The negative predictive value (NPV) of inconclusive ultrasound findings and low Alvarado score combined was 99.6%. Among children with inconclusive focused appendicitis ultrasound findings and an Alvarado score of 5-8, the NPV decreased to 89.7%.

CONCLUSION:

Children with inconclusive focused appendicitis ultrasound findings and a low Alvarado score are extremely unlikely to have appendicitis (NPV, 99.6%). Avoiding unnecessary CT of these patients is a safe approach to diagnosis.

 

When using CT for appendicitis, you need some peritoneal fat to identify the appendix.  So, very thin kids will be difficult to visualize the appendix on CT because they lack peritoneal fat.   However if there are no secondary signs of inflammation in the area of the appendix, that also makes appendicitis unlikely even if the appendix is not visualized.   To limit radiation exposure, you only need to do a pelvis CT.  Give IV and rectal contrast if the patient can tolerate them.   The cutoff for a normal appendix on CT is 7mm.

 

MRI can also be used to diagnose appendicitis. We are currently testing a MRI protocol for appendicitis.   The patient first gets scored on the pediatric appendicitis score.  If the score is 2 or more they are eligible for MRI testing.

 

Ultrasound is the first line test for identifying intussusception. 

 

Girzadas      ACGME Resident Survey Results

 

C Kulstad/Lovell    Preventing Diagnostic Error     Cognitive Bias

 

Misdiagnosis is the most common medical mistake and is a leading cause of malpractice claims. 

 

“Everyone is a human first and a novice or expert second”

 

There are 2types of causes for diagnostic error.  There are systems error and cognitive error.

 

 

*System and Cognitive Error

 

Error is more common when: uncertainty is high, the patient is unfamiliar to the physician, and the disease has an atypical presentation.   Cognitive error is a risk in high-pressure environments with many distractions. (the ER).

Nurses when acting as the wingman to the physician also need to be aware of their biases.  The nurse has the potential bias the physician either towards or away the right diagnosis.  If a nurse is biased against some aspect of the patient it can affect how the whole team approaches the patient. 

 

System 1 and System 2 Thinking.  To be a good EM Clinicianyou have to be able to switch back and forth between both systems.  Experienced clinicians have better developed system 1’s.  They have the experience to know when to switch into system 2 thinking.

 

*System 1 and System 2

Heuristics use pattern recognitions or mental shortcuts to solve problems.   Bias impacts heuristics a great deal.   This is a problem because ER docs use heuristics all the time in an environment that is prone to bias the heuristics.

 

Croskerry Six Biases:

1.     Over attachment to a particular diagnosis.  Anchoring on a specific feature too early and failing to adjust.

2.     Inheriting someone else’s thinking.  Following the cognitive path initiated by another provider.  This bias makes sign out a dangerous time.

3.     Failure to consider alternatives.  Tendency to stop looking once something is identified.

4.     Errors in estimation of prevalence.  Diagnoses that readily come to mind may be used more frequently.

5.     Bias related to patient characteristics or context.  Basically the physician has a personal perception about the patient and approaches the patient based on that concept of the patient.

6.     Errors associated with the clinician’s personality.  Over/under confidence is one example.   We need to have the appropriate level of both confidence and humility.

 

Tactics to overcome bias:

Cognitive debiasing:  Consider alternatives. Use metacognition or think about how you are thinking about a patient. 

Structured handovers:

Checklists :  Checklists help us to think of options we did not initially consider

QV&V: Qualify the source, validate the information, and verify the information.  Basically check that the info you are basing your decision on is correct and valid.

STEP Thinking: Story, Testing, Evaluate, and Plan

EMR Enhancements

Conference Notes 8-5-2015

E. Kulstad         Research Update

ACMC EM is #1 in the world in enrolling patients with intra-cerebral hemeorrhage  and hypertension into the ATACH 2 Trial!  Please continue to enroll these patients in the study.

The Macy Catheter can be used to give almost any medication rectally.   You can give up to about 500ml of water or pedialyte per hour rectally. This may be a therapeutic bridge option in the person who has a difficult IV access.

C. Kulstad       Study Guide Cardiology

There is no mortality benefit to routinely taking non-STEMI, ACS patients to the cath lab.   Thus, the patient with chest pain and bumped troponins and a non-STEMI EKG gets equal benefit from medical management as they would from going to the cath lab.

The absolute mortality risk reduction of ASA in ACS is 4%.  This data is derived from the 1988 ISIS study that showed ASA brought mortality down from 13% to 9%.  The absolute mortality reduction from heparin and LMWH in non-STEMI ACS is 0%.   The data for heparin and LMWH in STEMI is not known.  There is no Cochrane review on this topic.  We don’t really know the mortality benefit, if any, for heparin in STEMI.

POISE-2 study showed that you should hold ASA prior to abdominal surgery.  There is no increase in mortality if you hold ASA prior to abdominal surgery in patients with prior MI.  If you give ASA prior to surgery there is an increase in bleeding.  Elise comment: Most surgeons should not have a problem limiting surgical bleeding due to a patient’s ASA use.

electrical alternans

*Electrical alternans is a poorly sensitive and reasonably specific finding of pericardial effusion.

Stress echo is a moderately sensitive (80%) and specific (80%) test for coronary stenosis.   It however is not a good test to identify risk for major adverse coronary events (MACE)

A murmur at the right 2nd intercostal space is consistent with aortic stenosis.  These patients can present with chest pain, dyspnea and exertional syncope.   It is a murmur that is easy to miss.  The prevalence of aortic stenosis in the elderly is 3% (pretty high prevalence).  There is now a new option of transcatheter aortic valve implantation that allows for valve replacement therapy without thoracotomy.

pacemaker indications

*Indications for temporary and permanent pacemaker

Hi dose NTG has shown mortality benefit in CHF.   We probably under dose IV NTG.   One strategy to get a higher dose of NTG into a patient is to use sublingual spray.  Each spray gives 400micro grams per spray.  You can give a spray every few minutes in the hypertensive, pulmonary edema patient.

Tobacco cessation is the single most profound risk reduction strategy in patients with CAD.

Pericarditis pain is worse lying down.  Patients can also have pain with swallowing.

Burns/C. Kulstad      Oral boards

Case 1.  24 yo male presents with agitation and hyper-adrenergic toxidrome. Patient had to be restrained by EMS personel and was fighting against restraints.   His troponin and AST are elevated.   His CK is markedly elevated.  Urine tox screen is positive for cocaine.  Patient management of cocaine overdose required relatively high dose IV benzo’s for agitation and hyperadrenergic vital signs. Treatment for cocaine toxicity is benzodiazepines as your  first/second/third line.  Patient also required  IV fluids for rhabdomyolysis.  A bicarb drip for cocaine-induced rhabdomyolysis is controversial.  Cooling measures were also indicated to manage hyperthermia. Avoid beta-blockers in cocaine toxicity.  You can get unopposed alpha effects.

Case 2.  28 yo female with left shoulder pain.

ac separation

*Image

Diagnosis was AC joint separation.   Treatment is sling, pain control, and non-emergent orthopedic referral.  You have a two-week time frame to surgically repair an AC separation.   AC joint separation is the most common shoulder injury in athletes. High-grade separations require surgical repair.

Xray findings of AC separation: The acromion is not level with the lateral clavicle.

ac sepatration xray

*AC separation Xray

Case 3.  20 month old female brought in for crying.

svt peds

*Monitor strip

Exam shows enlarged liver.   Patient has cool extremities.

Diagnosis was SVT.  Treatment was IV adenosine 0.1mg/kg initial dose.  The rhythm slowed temporarily but then returned to SVT.  Second adenosine dose was 0.2mg/kg, which also did not convert the patient.  Child appeared more clammy/shocky so the next move was synchronized electrical cardioversion.  Patient was given intransal fentanyl prior to cardioversion.  Initial shock was 0.5j/kg.  Patient converted with first attempt at cardioversion.   If first attempt fails you can go to 1j/kg.

Remke    Pediatric SVT

Case 1.  7 day old with a heart rate of 300.   BP 75/47.   Afebrile.   Patient had been feeding well.  Lungs were clear.  Color was good.  Rythm Strip showed SVT.  Child was treated by peds cardiology in the ED with propranolol and rate was controlled.

Snip20150806_1.png

 *3 Types of SVT

To diagnose SVT  in pediatric patients, the heart rate should be great than 220 in an infant and >180 in a child.   There should be no P waves on EKG.  The rhythm should be very regular with no variability.  Sinus rhythm has more variability than SVT.   Notching of the T wave suggests sinus rhythm with the P wave buried in the T wave. 

svt vs sinus better

*Sinus Tach vs. SVT

svt algorithm

*Pediatric SVT Management Algorithm

Vagal maneuvers include: ICE to the face, Valsalva maneuver, and rectal stimulation with a thermometer.

For known WPW use procainamide instead of adenosine.  Elise comment: For orthodromic narrow complex SVT it is ok to use adenosine even if it turns out to be WPW.  It is the AHA recommendation. 

Avoid verapamil in infants and children.  It is contraindicated in patients under 1 year of age because it can result in V-Fib and hypotension.   It can be used as a second or third line therapy for SVT in teenagers.

After cardioversion, admit infants and discharge older kids/teenagers after a 2-4 hour period of observation.  If you identify WPW discuss with cardiology about admission for ablation.

Ede      Low Risk Chest Pain

Chest pain patients are considered low risk if the probability of major adverse cardiac events is 1-2%.

Patient’s response to GI cocktail or NSAID or NTG is not reliable for ruling in/out ACS.

Women may have more atypical symptoms.

The-HEART-Score.png

*HEART Score        Patients who are low risk by HEART score and have 2 negative troponins 3 hours apart have a less than 1% risk of MACE  (major adverse cardiac events)  in 6 weeks.

feb-2015-cp-shared-medical-decision-making-5-1.png

*Heart Score Decision Making

Hart        Tachyarrhythmias

Adult tachycardia algorithm

*Tachycardia Algorithm.   I would only add that procainamide can also be used and is probably preferred for wide complex irregular tachyarrythmias (WPW).

If you have IJ central line access, you can use a lower dose of adenosine because of a faster transit time to the heart.  Braden Parker noted that 3mg of adenosine worked quite well in a patient with SVT that he treated through an IJ line. 

Adenosine causes a sense of doom or chest pain in patients.  You should give some sedation prior to administering adenosine.

wpw

*WPW Syndrome

Avoid AV nodal blocking agents (betablockers, calcium channel blockers)  in antidromic (wide complex) tachycardias.  Blocking the AV node in antidromic tachycardia can result in death.

Aflutter usually has a rate right around 150.

a-fib-and-a-flutter-venn-diagram.jpg

*Atrial Fibrillation and Atrial Flutter

Calcium Channel Blockers have been shown to be superior to beta-blockers for Afib.

junctional tachycardia_digoxin

*Junctional Tachycardia can be hard to diagnosis.  Consider digoxin toxicity when you see Junctional Tachycardia.

VT

*Diagnosing V-Tach

Snip20150806_2.png

*Girzadas note: Another Approach to Diagnosing V-Tach on ecgpedia.org. I have always struggled with differentiating VT and SVT with Aberrancy.  This seems like a simple way to do it.  In lead II if the initial upstroke or downstroke of the QRS takes more than a box it is likely VT.  A caution, It is not validated yet. 

First line treatment for Stable Ventricular Tachycardia is Procainamide.   Second line is Amiodarone.

torsades

*Torsades      Treat with Magnesium or Overdrive Pacing

Parker         PodCast Pearls  for Codes

Traumatic Arrest is usually due to one of a few things: exsanguination,  pericardial tamponade, or tension pneumothorax.    Based on that, is there any reason to do CPR for traumatic arrest?  CPR can also hinder other activities/procedures being performed on a trauma patient. CPR can increase the risk of blood exposure to the resuscitationist.

Mnemonic for PEA in the Trauma patient:  HOTT=  Hypovolemia, Oxygenation, Tamponade and Tension Pneumothorax.

C Kulstad and Girzadas comments:  We are uncomfortable dropping CPR completely from trauma resuscitation.  We think there is medical-legal risk to that.  Possibly a more middle of the road approach would be to prioritize life saving procedures.  If CPR is hindering other life saving procedures such as gaining IV access, relieving tamponade,  or relieving tension pneumothorax stop CPR for as short a time as possible and perform the procedure and re-start CPR as soon as possible.

PEA Management:  Use ultrasound to identify causes.  If the QRS in PEA is wide think metabolic causes like hyperkalemia, TCA overdose, infarction.  If the QRS is narrow think obstructive causes such as tamponade, tension pneumo, or PE.

Jamieson        Parasites

A parasite lives at the expense of the host.

The CDC Lists 5 Parasitic Infections we should know.

Chaga’s disease: Caused by protozoan Trypanosomi Cruzi.  Acutely, patients have fever and malaise.  They can also acutely have eyelid swelling, myopericarditis, and meningitis.  Chronically patients can have CHF or megaesophogus.   You need to get the treatment drugs from the CDC.

chagas-disease-bug.jpg

*Eyelid edema from acute Chaga’s Disease

Cystercercosis:  You get the worm from eating infected pork. Neurocystercercosis can result in seizures.   When the infection is in the muscle if feels like bumps.   Treat with albendazole, anti-seizure meds, steroids, and surgery.

neurocystercircosis

*Nueorcystercercosis

Toxocariasis: Caused by ingestion of cat or dog feces. Acutely the patient will get a hepatitis or pneumonitis.  The pneumonitis can look like asthma.  Kids can also get unilateral visual impairment from fundal involvement.

Toxoplasmosis: Caused by ingesting cat feces.   Infection can be spread vertically from mom to infant.  Congenital toxoplasmosis can cause spontaneous abortion, fetal death, and fetal neuro deficits. Immunocompetent adult patients can have a mono-like syndrome.  Immunocompromised adult patients can get encephalitis.  

Trichomoniasis:  Most common non-viral STD.  Can be asymptomatic for months.  Women have vaginal symptoms.  75% of men are asymptomatic.  Can result in pregnant women having premature rupture of membranes and preterm delivery.

Conference Notes 7-29-2015

If you don't see images scroll to the bottom and click "Read in Browswer"

Paquette      Study Guide     Special Patients

Black tar heroin mixed with lemon juice may harbor fungus that when injected can result in fungal chorioretinitis.

50% of elderly patients hospitalized after a fall die within a year.   Falls represent a sentinel event and are not a normal part of aging.

trench-foot-pictures-6.jpg

*Trench Foot is stratum corneum breakdown due to continuous water exposure.  This is commonly seen in homeless persons forced to wear wet shoes and socks for prolonged time periods. Treatment is dry socks and dry shoes.

Treat maggots with ethyl chloride.  The topical Freezing Spray (ethyl chloride spray) works well to kill maggots.   A recent study showed that a yankaur suction device works well to remove maggots.

To optimally visualize the airway in obese patients, use RAMP positioning.  The goal is to line up the external auditory meatus with the sternal notch. 

ramp2.png

*RAMP positioning

Cotton fever is due to a gram negative rod that exists on cotton balls. This gram negative rod can get into heroin when the cotton is used as a filter.   The fever is due to an exotoxin from the gram negative rod and is self-limited.  No treatment other than antipyretics is indicated for cotton fever.  However, this provides little help because it will be hard to differentiate cotton fever from sepsis when the patient first presents to the ED.

The right heart valves are more prone to endocarditis than left side heart valves.  Injection cocaine use is more likely to cause endocarditis than injection heroin use because cocaine has a shorter half life resulting in more injections and cocaine causes direct damage to the valves.

Obese patients tend to have EKG’s with low voltage and inferior t-wave flattening/inversions.  These changes are thought to be due to increased chest wall thickness and possible shift of the normal axis.

Trauma patients over age 50 account for 30% of Trauma deaths.  Trauma patients of advanced age have less physiologic reserve to withstand severe injury. 

Subdural hematoma is more common in elderly patients.

sub vs epi

*Subdural vs. Epidural Hematomas

In patients over 80 years of age, chest pain is not the most common symptom of AMI.  Dyspnea and weakness are more common presenting complaints.

Downs syndrome patients have chronic atlanotaxial instability

aa instability

*Atlanto-axial Instability

“Parents have brought a child with Down’s Syndrome ‘walking funny’ to an emergency department for evaluation,” Dr. Bull recalled. “And then the ED would X-ray the child’s hips and say nothing is wrong. But they were not aware of the importance of evaluating the cervical spine.”

The neurologic manifestations of symptomatic AAI include easy fatiguability, difficulties in walking, abnormal gait, neck pain, limited neck mobility, torticollis (head tilt), incoordination and clumsiness, sensory deficits, spasticity, hyperreflexia...and (other spinal cord) signs and symptoms. Such signs and symptoms often remain relatively stable for months or years; occasionally they progress, rarely even to paraplegia, hemiplegia, quadriplegia, or death. Trauma rarely causes the initial appearance or the progression of these symptoms. Nearly all of the individuals who have experienced catastrophic injury to the spinal cord had weeks to years of preceding, less severe neurologic abnormalities..."

During intubation of a Down’s patient, Great care must be taken to maintain the

neck in a neutral position. This may be accomplished

by placement of a soft collar after induction of anes-

thesia to avoid extreme neck flexion, extension, and

rotation. Leaving the collar in place may also serve as

a valuable reminder to all caring for the patient dur-

ing the perioperative and postoperative period that

cervical instability may exist

autonomic dysreflexia

*Autonomic Dysreflexia can be seen in spinal cord injured patients who have a uti, constipation, pressure ulcer or constricting clothing below the spinal cord level of their injury.  The patients don't feel discomfort from these problems but develop HTN, flushing, sweating, and headache. 

Asokan     Interpreting the Chest Xray

We are responsible for everything on the X-ray, not just the aspects that we are interested in.  Be sure to look at the neck, axillae, and abdominal areas of a CXR.

Pathologic densities in the lung show up as a white density signifying abnormal fluid.

Normal-CXR-Labelled.jpg

*Normal CXR

If you see an abnormality, look for a second or third less obvious abnormality.  Your eyes get stuck on the most obvious abnormality.

Consolidation is when the alveolar space is filled with inflammatory exudate.  Consolidations have air bronchograms.

Consolidation1_big.jpg

Consolidation with air bronchograms

pleural effusion

*Upright film shows a pleural effusion with a curved upper meniscus.

When looking for pneumothorax, look between the ribs for the pleural line.

cxr_ptx_1.png

*Pneumothorax. Note the pleural line between the ribs on the right.

CHF

*CHF Findings

peribronchial cuffing

*More views of peribronchial cuffing

sbo

*Small Bowel Obstruction

coffee bean

*Coffee Bean Sign of Volvulus

Jamieson     M&M

Life Lesson #1:  Don’t use dermabond around the eye.   If you need to disolve glue on an eyelid or your finger gets glued to the skin,  use erythromycin ointment.   It doesn’t break glue down immediately but it will over a relatively short time.

Life Lesson #2:  In order to figure out  what is going on with non-verbal nursing home patients, it may require a call to the nursing home.   Steve described the case of a nonverbal nursing home patient who otherwise was well appearing. He called the nursing home and learned that the patient was normally quite talkative. He proceeded to CT scan the patient and identified an intra-cerebral hemorrhage.

Life Lesson #3: Beware the back hallway bias.  This anchoring bias can cause you to downplay a patient’s acuity and symptoms.   Just because a patient is in the hallway and not in a critical care room doesn’t mean they can’t have a critical illness.

Life Lesson #4: Intubating obese trauma patients can be risky.  Expect O2 desaturation to occur more quickly than expected.  Be sure you have maximized your  pre-oxygenation.  Place the patient in RAMP position.  Use high flow continuous nasal oxygenation during the pre-oxygenation and intubation periods.  Have video laryngoscopy ready as either your first device or back up device.  Have an LMA ready to use as a rescue device.   Be sure to check that your oxygen source is working properly.  You can use ketamine as an induction agent to perform delayed sequence intubation; you can preoxygenate and evaluate the airway prior to neuromuscular blocking.

Htet/Meyers/Faculty       Orthopedic Exam Lab

Conference Notes 7-22-2015

A special Thank you to Christine Kulstad for her help with the Conference Notes this Week!

Lovell/Htet        Oral Boards

Critical Actions Case 1:  Septic Shock from Capnocytophaga canimorsus/dog bite

--Identify Sepsis

--Rapid IVF, early broad spectrum antibiotics including dog bite coverage

--Elicit history of dog bite

--Reassessment

--ICU admission

(also consider Xray, irrigation of wound, consider tetanus update, rabies)

Teaching points:  Capnocytophaga canimorsus

--fastidious GNR identified in 1976, likes to eat iron

--normal oral flora of kitties and doggies

--Risk groups:  asplenic, alcoholics, beta thal, smokers (high iron), immunosuppression

--Alcoholics have high blood iron + immune comp.

--WORST OUTCOMES in asplenics:  high iron + decreased phagocytosis; high M/M!!

--Symptoms usually within several days

--Fever, V/D, malaise, abdominal pain, myalgia, confusion, dyspnea, headaches, rashes

--Special culture media

--Tx:   Dogmentin!  so use IV Unasyn, also susceptible to third gen. Cephalosporins

Critical Actions Case 2:  Submassive to Massive PE

--Recognize risk for PE (travel)

--Diagnose PE

--Risk stratify (Echo, BNP, Trop)

--Recognize decompensation and need for thrombolytics

--Assess contraindications to tPA

--ICU admission, Intensivist consultation

Teaching points:   Submassive/Massive PE

--Submassive:Normal BP but RV dysfxn (Echo or BNP) or myocardial necrosis (Trop)

--Massive: Low BP for 15 minutes or code or resp. failure or need for pressors

--Thrombolytics for massive, case by case for submassive

--r-tPA dose 100 mg IV over 2 hours

--Code dose, 50 mg bolus (maybe + 50 mg)

--start IV Heparin when PTT < 2X nml

--Lytics in PE:  2% ICH risk, 6% major bleed/transfusion

Critical Actions Case 3:  Jellyfish envenomation

--Identify as Jellyfish envenomation

--Irrigate 

--Consider vinegar, hot water

--Monitor for systemic sx, allergic sx

--Treat with benadryl, consider steroids, analgesics

Teaching points: Jellyfish Envenomation

--Mechanism-tentacles with venom containing nematocysts

--Symptoms--Itch, swell, burn, tingling/numbness.  Severe with N/V, HA, weak, dizzy, SOB

--Treatment-irrigate with NS or sea water (fresh water makes worse), remove possible remaining nematocysts by shaving skin or scraping skin

--Use benadryl, consider steroids, give analgesics

--vinegar-prevent further nematocyst firing, hot water to breakdown venom-some controversy over effectiveness

Levato     Pharmacy Update

Community-acquired but complicated UTI- E coli, Klebsiella, Proteus most likely. We have been using ceftriaxone for this but it is a broad spectrum antibiotic that is causing c diff in our hospital. Cefazolin found to be 89% sensitivity in E coli, 90% in Klebsiella, 100% Proteus. So new guidelines are to use cefazolin for these patients. CAUTION- not appropriate for nursing home patients. OK for pyleonephritis.

Ketaneh  (Class of 2014)  Starting out as a New Attending

Looking for a job: Take some pressure off- its just your first job, you can change it later. He interviewed all over and recommends it. Prospective employers will usually pay for your interview expenses so don’t shy away from multiple interviews. It helps to see a lot of practice environments too, see where you fit in.  Use the faculty resources here (they know somebody everywhere).

You will have a lot more money- try to live like a resident and save for a while. Student loans will come due, 401k should be funded. Need to find your work-life balance- how many shifts due you want to work to pay for your lifestyle.

Starting a new job- get familiar with the EMR before your first real shift. Will save you tons of time and pain before your first shift.

Have your charting macros prepared before you start.

Know where you are going to look stuff up.

Be super nice and respectful to everyone.

Get your charting done so you stay off the medical records radar. 

Get to know the culture of your workplace: protocols, antibiotic choices, sedative choices.

Nick told a scary story of how he ordered a fentanyl drip and the pharmacists and nurses were not familiar with using fentanyl as sedation and dosing errors were made.

At most ED’s you at times will be working alone with a couple of nurses.

Be very clear when communicating with nurses, they will do exactly whatever you say.

An important part of our job is charting and billing.  It’s just a fact of life.  Your income and job security depend on it.

ups and downs

*Ups and downs.   You have to be able bounce back from the downs as an ER doc and temper/be humble with the Ups

There are other jobs out there.  You can change jobs.   It is important to protect your reputation so people think well of you and you keep the door open to other opportunities.

Be smart with your money.   It is OK to rent for awhile until you have a clear idea of your monthly income.  Then live below your means so you are funding your retirement properly.

As a resident and attending, going the extra mile will not go unnoticed.

Get your paper work done. 

Favorite FOAM websites:  lifeinthefastlane.com, aliem.com, emcrit.com

The goal for an ED doc in the community is to not be ultra cutting edge and not be practicing medicine from 20 years ago.  You want to be somewhere in the middle and up to date. 

Regan       Seizures

causes of seizures

*Causes of Seizures

Generalized Seizures: Affects both Cerebral Hemispheres. There is loss of consciousness.

siezuretypes.gif

*Seizure Types

Partial Seizures include: Simple in which conscious is maintained and Complex in which consciousness is altered.

Initial questions to ask the patient:  Do you have a seizure disorder?     Is there anything that would have lowered you seizure threshold?  (sleep deprivation, fever, etoh use, missed medication, stress)

EKG can be a quick initial test to pick up electrolyte abnormalities causing seizure.

Seizure meds

*Medications for seizure management

Elise comment: In the Peds ED there is frequently the availability to get an EEG rapidly for a child with a new onset seizure.  This rapid turn around of an EEG can be very helpful to neurology to make good management decisions for the patient.

Case 1.  Etoh withdrawl seizures can be treated with Ativan first line and Phenobarbitol second line for persistant  seizures despite Ativan.    Christine comment: You can push the upper dosage range of Ativan in the ETOH withdrawal patient. Their GABA receptors are down regulated.   Elise comment: If your Ativan is not working in any type of patient, start thinking about underlying causes such as hypoglycemia, hyponatremia, INH poisoning, intracranial hemorrhage.

20mg/kg loading dose works for Phenytoin, fosphenytoin, Phenobarbital, Valproic Acid, and Keppra.  Basically 20mg/kg works for all commonly used epileptics other than Ativan.

Check the patient’s tongue for lacerations and hematoma.  Serious tongue injuries  could pose an airway obstruction issue.

Neuro exam in infants

Look for:

Balled fists with thumbs tucked in, this is an upper motor neuron sign (Babinski sign in infants)

Altered/increased tone

Abnormal Reflexes

Eye deviation

Elise comment: Midazolam IM is a very rapid acting anti-seizure medication that is useful in kids.   Eye deviation is an important finding in kids to identify a seizure.

30-50% of patients with pseudo seizures also have epileptic seizures.  This makes diagnosis and management possible psuedoseizures very difficult. 

Carlson                      Antidotes

antidote-chart-forcpcs2008-1-728.jpg
antidote-chart-forcpcs2008-2-728.jpg

*Antidotes

Isopropyl alcohol ingestion is not an indication for fomepizole.  It is metabolized to acetone.

Lead has 3 potential antidotes: BAL,  EDTA, and Succimer.   Peanut allergy is an absolute contraindication to BAL therapy.

Gyromitra mushrooms look like a brain and can cause seizures.  The antidote is pyridoxine (also the INH antidote/INH also causes refractory seizures).

gyromitra mushroom

*Gyromitra mushroom

Sodium Bicarbonate is an antidote for TCA poisoning.  The sodium overcomes Na Channel blockade.  The Bicarbonate decreases free TCA.   Andrea prefers bolus dosing with 1-2 ampules while closely observing the patient and the monitor.

Calcium channel overdose can cause bradycardia, hypotension, and hyperglycemia.  The antidote is the “perfusion salad”: calcium chloride, glucagon, intralipid, atropine, and insulin euglycemic therapy.

Deadly nightshade contains bella donna toxin.  Bella donna can be thought of as a Benadryl overdose.  Both cause an anticholinergic toxdrome.  The antidote for anticholinergic toxidromes is physostigmine.   Physostigmine should not be used in undifferentiated coma or in mixed overdoses.  However, it is safe in pure anticholinergic toxidromes.  

deadly nightshade berries

*Deadly nightshade berries

deadly nightshade leave

*Deadly nightshade leaves  (note the little “ears” small leaves around the big leaf)

Paint stripper that contains methylene chloride if ingested will be metabolized to CO.  Methylene chloride gets metabolized by liver to CO and can cause CO toxicity.    Antidote is hyperbaric oxygen. 

Statistically the most common cause of CO toxicity in the summer is indoor grilling or using a grill on a screened-in porch. 

Guidelines For Hyperbaric Oxygen Therapy In Carbon Monoxide Poisoning Emergency Medicine Practice.JPG

*Hyperbaric indications for CO Toxicity

Causes of cardiac glycoside poisoning: digoxin, oleander, foxglove, Lilly of the valley, and the practice of toad licking.   Antidote is FAB fragments

West      Fever in Kids

Rectal temperature measurement is most accurate.   The temperature that determines a fever is 38C.

fever algorithm under 3 months

*Fever algorithm for kids </= to 3 months

For febrile kids (with immunizations up to date) over 3 months of age with no source on physical exam the next best diagnostic study is to check the urine.

Brain damage is not a complication of fever due to infection.

Faculty comment: Parents frequently under-dose Tylenol and Ibuprofen.  Check the dosing parents are giving to kids if the antipyretic is not bringing down the fever.

Harwood comment:  You may not need to do an LP on febrile kids age 30-60 days old if they have the following:

Well appearing child

Normal urine

Normal ANC

Normal Procalcitonin

This decision rule has not been validated. 

Elise comment: Right now there is a grey zone (based on age of infant) of whether there is a need to do an LP.  There is reasonable practice variation in non-toxic appearing, febrile kids age 30 days to 60 days.   Most faculty have a cut-off at 6 weeks, but up to 8 weeks is reasonable.  Ill appearing kids at any age require LP.

fever management table

Fever Algorithm for ages Newborn to 36months

Bonder     Patient Satisfaction

Patient satisfaction rates are a factor in medicare reimbursement.

Research has shown that patient satisfaction measurements are biased and don’t correlate with clinical quality metrics.   Also, mortality rates tend to be higher in hospitals with high patient satisfaction levels.

Safety net hospitals and teaching hospitals regularly perform more poorly on patient satisfaction measurements.   There are geographic variabilities as well with California and East Coast hospitals usually scoring lower than other regions.

4 habit model

*Kaiser 4 Habits Model from Kaiser Health

Elise comment: Patient satisfaction measurements are not based on your clinical care or diagnostic accuracy.  They are based on how you communicate with the patient.

Christine comment: At the completion of the ED visit, take the time to explain to the patient what the tests showed and how they ruled out dangerous diagnoses.

Patient satisfaction measures

*Patient Satisfaction Measures

Conference Notes 7-15-2015

If you don't see images, please scroll to the bottom and click "Read in Browser"

Burns     EKG Basics

 

*EKG Intervals

Harwood comment: when determining the QRS interval, measure at the lead with the widest QRS.

General Approach to Evaluating an EKG

  1. Heart Rate
  2. QRS: wide or narrow?
  3. Rhythm: Regular or Irregular?
  4. P waves
  5. Axis   If the QRS is predominantly positive in I and II or AVF it is a normal axis
  6. ST changes
  7. LVH, QT interval
  8. Special signs (osborne wave, brugada sign, WPW)

 

*Rule to determine Axis

 

*Simplified LVH Criteria

 

Ianitelli     Procedural Sedation

 

*Comparison of Common Sedation Agents

 

*Comparison of Succinylcholine with Rocuronium

 

Case 1.   19yo male with GSW to abdomen.  Patient is hypotensive and tachycardic.  Barounis comment: I would use low dose etomidate to avoid hypotension.  Give a higher dose of succinylcholine due to poor perfusion of muscles in shock.   Elise comment: I would use ketamine to avoid hypotension.   Harwood comment: I would use reduced dose of etomidate.

 

Case 2.   32 yo female with altered mental status.   Hypotensive, tachycardic, and febrile.   O2 sat is 92% on RA.  

Harwood and Girzadas comment: Delayed Sequence Intubation with ketamine. Pre-oxygenate then give succinylcholine to intubate.   ED Pharmacist comment: Have push dose pressors ready for any hypotension that would develop.

 

Case 3.   72 yo female found down. Patient has right hemiplegia.  Patient has been on the floor for at least 18 hours.   BP is 100/60,  Pulse of 70.

 Consensus was using ketamine or low dose etomidate to avoid hypotension.  Consensus was using rocuronium to avoid hypokalemia in a person with possible rhabdomyolysis/hyperkalemia/renal failure.    Samir Patel comment:  Succinylcholine is safe in chronic renal failure patients but be prepared to manage any hyperkalemia.  In the setting of acute renal failure, rhabdomyolysis, or hyperkalemia, avoid succinylcholine.

 

*Contraindications to Succinylcholine

 

Case 4.   58 yo male with a Gluteal Abscess.  Normal vital signs.   Most people felt ketafol would be the optimal choice.  Playing devil’s advocate, Kelly Williamson felt that using pain control and light sedation with fentanyl and versed could also work.  The benefit of this approach would be to avoid procedural sedation and any complication of sedation.

 

Kennedy    Use of Pressors

 

*Pressor Chart

 

Norepinepherine is your workhorse pressor.  It is probably the first choice for most non-hemorrhagic shock situations.

 

Septic shock:  first choice norepinephrine, second line is epinephrine or vasopressin

Anaphylactic shock: First choice is epinephrine,  second line is vasopressin

Spinal Shock plus bradycardia: Epinephrine,  second line is dopamine

Spinal Shock with normal rate:  First choice is norepinephrine

Cardiogenic shock:  First choice:  Use a combination of dobutamine for inotropy with norepinephrine to reach a MAP of 65. 

Pediatric Septic Shock (cold extremities)  First choice is epinephrine,  second line is dopamine

Pediatric Septic Shock (warm extremities)  First choice is norepinephrine

Hypoplastic left heart syndrome with shock: First choice: Milrinone   Second line: Dobutamine

 

Push dose pressors:  Give during RSI in a hypotensive patient.   Can also use in other clinical situation with expected rapid/short duration episode of hypotension

Epinepherine 1ml of cardiac epinephrine  added to 9ml of saline in a syringe.  You can give 1 ml of this diluted epinepherine every 2 minutes as needed for hypotension.

 

Myers     Ventilator Management 

 

Ventilator management in the lung injured patient who doesn’t have asthma or COPD:   Start on assist control.  Keep volume low, tidal volume of 6 ml/kg of ideal body weight.  Use the ventilator rate to provide appropriate ventilation.  Don’t change tidal volume to adjust ventilation.  15 breaths per minute is a reasonable starting point.  Start FIO2 at 100% and rapidly titrate down to FIO2 of 40% if possible to hit goal of O2 saturation >90%.  You can also use PEEP to improve oxygenation.  PEEP stents open alveoli to improve oxygenation.  PEEP can negatively impact the patient’s preload resulting in hypotension.   Keep plateau pressure less than 30.  Plateau pressure is the measure of risk of barotrauma.  It is a measure of the alveolar barostress.

 

*Suggested FIO2/PEEP Table   

 

*ARDSnet Protocol

 

Obstructive Strategy  (Asthma, COPD Ventilator Management):

Lower the patient’s  respiratory rate to a rate of 10 or even 8. Increase expiratory time by setting the I:E ratio to 1:5.  Permissive hypercapnea is OK.  Keep ph >7.2  Set PEEP to 0.

 

*Obstructive Ventilator Strategy

 

In the crashing intubated asthmatic patient, your first move is to disconnect the ventilator.  Then compress the chest manually to improve exhalation of air. 

 

Htet    Introduction to Codes

 

Have the same approach every time to every Code you manage.

Start with IV, O2, Monitor, Vitals, Dexi, EKG,

Get help from nurses and techs, respiratory therapist, and other physicians.

Proceed in your management based on the data you gather from these first two steps.

 

Case 1.  64 yo female who is unresponsive following an mvc.

 Start with IV, O2, Monitor, Vitals, Dexi, EKG,

Get help from nurses and techs, respiratory therapist, and other physicians.

EKG shows TCA toxicity. 

Consider intubation to potect airway

Give IV fluids

Consider pressors if hypotensive.

Give IV bicarbonate to narrow the QRS, and improve cardiac output and blood pressure.

 

*TCA EKG  Note tall, wide terminal R wave in AVR and Wide QRS diffusely

 

Case 2.  65 yo female with fever and cough, hypotension and tachycardia.

Suspicion is septic shock. 

Start with IV, O2, Monitor, Vitals, Dexi, EKG,

Get help from nurses and techs, respiratory therapist, and other physicians.

Intubate the patient considering the adjuncts of push dose pressors or delayed sequence intubation.

Give IV fluids, start antibiotics, give pressors if patient’s BP is not volume responsive.

Get appropriate labs including blood cultures and lactate.

 

Harwood comment: Bipap has been shown to decrease intubations but data demonstrating that it decreases mortality is difficult to find.

 

Case 3. 80 you female with sudden right hemiplegia. 

Start with IV, O2, Monitor, Vitals, Dexi, EKG,

Get help from nurses and techs, respiratory therapist, and other physicians.

Get rapid CT of head

Identify last known normal neurologic function

Perform NIH stroke scale

Give TPA if indicated

 

Samad    Intro to Renal Ultrasonography

 

*Anatomy of Kidney on U/S 

 

*Chronic Kidney Disease usually demonstrates smaller than normal kidneys, and decreased sonographic differentiation between cortex and medulla.

 

*Ureteral Pelvic Junction kidney stone

 

*Mild Hydronephrosis   The key to identify mild hydronephrosis is seeing hypoechoic areas within the normally hyperechoic renal sinus.

 

CT is better than U/S for evaluating for pyelonephritis and it’s complications.

 

The first three differential diagnoses for enlarged kidney are:  DM, multiple myeloma, and HIV nephropathy.  The fourth is an infiltrative process like amyloidosis, lymphoma, or leukemia.

 

There was a discussion of a ureteral jet seen on ultrasound.  A ureteral jet is a sudden rapid flow of urine from the ureter.  If the ureteral jet is not seen, it suggests a significant ureteral obstruction.  Elise comment: If you have a ureteral jet on one side and not the other it suggests a complete obstruction of the side without the ureteral jet.  The finding of a unilateral absent ureteral jet may be a useful marker when trying to convince consultants to place a percutaneous nephrostomy in a patient with a suspect obstructed/infected kidney. 

 

 

*Ureteral Jets on U/S using the Doppler U/S

 

Levine     Understanding the Body’s Response to Traumatic Experiences

 

Sometimes the great care we strive to give patients in the ED can result in psychological trauma to those same patients despite our best efforts.

 

Psychological stress/trauma can result in real physiologic responses (fight or flight response)

 

Co-Regulation speaks to how caregivers can help a person experiencing emotional or psychological trauma cope with that event.   We can control our tone, pace, and volume of our voice to calm and reassure patient.  Gentle, appropriate touch and pleasant facial expressions can also calm and reassure the patient.

 

Physician strategies to diminish psychological trauma to the patient:

Allow family/friends to be with the patient

Educate patients about their diagnosis and what to expect both in the ED and when they go home.

Call Patients by their name and speak to them, not about them.

Remain calm and make a conscious effort to monitor your tone, volume, and rhythm of voice.

Use light touch on the arm, hand or shoulder  to calm the patient.

Use verbal therapy.  The following are a few examples:  We are going to take care of you.  We are going to make you feel better.  You can say:  I am here for you.  We are going to get you through this.   Life is really hard.  You have been through some really tough stuff.   Validate what patients are feeling and be careful to not undermine their thoughts, fears, or concerns.  

 

Bonaguro     Sepsis Update

 

 

Sepsis continuum

 

When you suspect sepsis tell the nurse or SIMs that the patient is a Code Sepsis

Get an initial lactate and a 6-hour lactate

Septic Shock requires 30ml/kg crystalloid bolus.  Give an initial 2 liter bolus instead of a 1 liter bolus.  If you are worried about causing CHF give 2 liters in 500ml aliquots and frequently re-assess the patient.

Do a 6 hour re-exam including vital signs, cardiopulmonary exam, cap refill exam, pulse evaluation and skin exam.   OR  Do a bedside cardiac ultrasound and passive leg raise test.  OR Place a central line and get a CVP.

You must write your sepsis H&P note within 6 hours of the patient’s ED arrival.

An EMR fix for these new requirements is the “ED Sepsis Advisor Power Plan”

This is a more advanced power plan in FirstNet than the ED Sepsis Power Plan.   The Sepsis Advisor Power Plan has most of the above testing and documentation requirements built in.  

 

 

 

 

 

 

Conference Notes 7-8-2015

If you don’t see the images, please scroll to the bottom and click on

“view in browser”.

Lambert   Ultrasound Physics Intro

 Low frequency sound waves travel further than high frequency sound waves.  

Low frequency probes have less image quality than high frequency probes but they can visualize much deeper structures.

High frequency probes have low penetration but better picture quality in the more superficial tissue.

 

Near Field is the image area on the screen closest to the probe.  Far Field is the image area on the screen further away from the probe.

In all probes, the near field images are usually better than the far field images.  There is more sonic information coming back to the probe from the near field than the far field.  

 

Image Orientation:  The probe indicator should correspond with the indicator on the screen.  In general for all imaging except echocardiography, the probe indicator should be pointing either to the patient’s head or the patient’s right side.  When performing echo’s, the probe indicator is oriented to the patient’s left side. 

 

 

*Shadowing is a high attenuating artifact.  Shadowing from a gallstone.

 

 

*Low attenuation  structure results in an enhancing artifact (more echogenic distal to the structure).  There is also a refraction artifact in this image causing an edge shadow.

 Set your image depth so that the structure you want to image takes up 7/8 of the screen.   If the image of the organ of interest  takes up less screen space than that you are wasting the real estate of your screen.

Set your gain so that the image echogenicity is consistent from top to bottom of the screen.

Always scan a structure in two planes.   Identify the boundaries of the structure you are interested in.

 

Lambert    7UP  Scan

 

Mike credited Chris Kerwin with inventing the term 7UP scan.

The 7UP scan is a rapid ultrasound evaluation for the patient in shock.

It is a similar protocol to the RUSH (Rapid Ultrasound in Shock) Protocol.  

Editors note: It was easier for me to grab some images of the RUSH protcol off the internet then get 7UP images.  My apologies to Mike and Chris.

 

 

*RUSH Protocol

 

 

*RUSH   Algorithm

 

The 7UP  protocol also images the RUQ,  Subcostal area, Abdomen, Pelvis, then Chest.

 

In a hypotensive patient with abdominal pain, fluid in Morrison’s pouch and a positive UCG there is nearly a 100% likelihood of ruptured ectopic pregnancy.

 

*Cardiac Tamponade with RV compression.  Note the scalloped/compressed appearance of the RV.

 

*Type A Aortic Dissection

 

*Ultrasound consistent with PE.  Typical RV is triangular in shape and smaller than LV.  PE with increased right side pressures will show a proportionately larger RV.

 

*IVC in Hypovolemia

 

*IVC in Hypervolemia

 

*AAA with Thrombus

 

When looking for pneumothorax, image 3 sites on each hemithorax.  If you don’t see normal sliding of the pleura, a pneumothorax is present.

 

 

*Ultrasound Imaging of Pneumothorax.  The left image is normal lung that shows pleural sliding and hyper-echoic comet tails.  The right side image is a pneumothorax where pleural sliding and comet tails are absent. 

 

Lambert/Frazer/Chastain/Chan and other Team Ultrasound Members             Ultrasound Lab

 

 

 

 

 

 

 

 

 

Conference Notes 7-1-2015

BG:  ph=7.0, PO2=200, PCO2=30, HCO3=5.   FIO2=100%  O2 on ven

If you don't see images scroll to the bottom and click on "read in browser"

Jeziorkowski/Girzadas  Oral Boards

 

Case 1.  55yo male presents in coma.  The patient is found to have an anion gap metabolic acidosis and an osmolal gap.  Urine shows calcium oxalate crystals and EKG has prolonged QT interval.  Patient was diagnosed with ethylene glycol toxicity.  Patient was intubated for airway protection.   Fomepizole was given to block alcohol dehydrogenase.   Nephrology was consulted for emergent dialysis.  Optimal care included a bicarb drip and thiamine and pyridoxine co-factors given to optimize metabolism of glyoxylic acid.   

n  Anion Gap = Na-(CL + HCO3) and should be 15 or less

n  This Case: 140-(105 + 5)=30

 

 CAT MUDPILES mnemonic for anion gap acidosis

 Cyanide, CO, CHF

 AKA, Aminoglycosides

 Theophylline, Toluene, Tylenol

 Methanol

 Uremia

 DKA

 Paraldehyde, propylene glycol

 Iron, Isoniazid, Inborn Errors

 Lactic acidosis

 Ethylene glycol, Ethanol (lactate)

 Salicylate, Starvation Ketosis

 

Case 2.  7 yo male shot in right thigh during a drive-by shooting.  Patient was resuscitated with IV fluids 20ml/kg  and IV PRBC transfusion of  20ml/kg.   Patient did not have hard signs of vascular injury.   Patient did have abnormal ABI of right lower extremity.   CTA of right thigh showed vascular injury.  Patient went to OR. 

 

Hard Signs of Vascular Injury mandate operative repair

Pulsatile bleeding

Pulsatile hematoma

Bruit/thrill

Absent pulse distal to injury

Ischemic signs (pain, pallor, coolness, paralysis)

 

EAST Guidelines

Hard signs should prompt surgery without angiogram

ABI <0.9 or other soft signs should get  CT angiography

CT angiography is the primary diagnostic study

No Hard/soft signs and normal ABI can be discharged

 

Case 3. 4 yo male fell in pool and was submerged for 1-2 minutes.  When rescued, patient coughed and started crying.  There was no loss of consciousness and no required resuscitative efforts.  On arrival to the ED , the patient had normal mental status,  normal vital signs, clear lungs, pulse ox =97% on room air and CXR was normal.  Patient was observed in ED for 4-6 hours and remained asymptomatic. Patient was discharged home. 

  • A patient that was immersed in a liquid and suffered any sort of respiratory event is considered to have suffered a drowning
  • Asymptomatic patients with clear lungs, normal pulse ox, and normal CXR can be discharged after 4-6 hours of observation.
  • Criteria for Admission(altered mental status, abnormal vitals,  P/O<95, respiratory symptoms, abnormal lung sounds, abnormal CXR)

 

 

Htet    M&M

 

There are studies to document that morbidity and mortality in teaching hospitals increases during July and August.  It is a real phenomenon.

 

Case: 65 yo male presents with hematuria.  He is also hypoxic and hypotensive.   Patient is on warfarin for atrial fibrillation.

Labs showed an INR of 7 and a lactate of 5.   WBC= 13.6.  HGB=8.  CXR showed a RLL infiltrate and effusion and a possible small right side apical pneumothorax.

 

FEIBA and vitamin K was given.  Patient was also treated for pneumonia/sepsis with Ceftriaxone and Azithromycin.

 

Later that day, patient dropped his pressure and became hypoxic.  He deteriorated further into a PEA arrest.   A chest tube on the right was placed during resuscitation out of concern for possible worsening pneumothorax and pus came out in the tube.  

Diagnosis was empyema and warfarin coagulopathy.

 

Dr. Htet’s Teaching Points:

  1. Intubate early in course of illness
  2. Consider CT of Chest to further evaluate the lung findings of pneumonia/effusion.  
  3. Consider a chest tube early if empyema is in the differential diagnosis.
  4. Pursue the cause of shock aggressively.  Elise comment: An ABG can be useful to help determine the cause of shock.   The Emergency Physician should set for themselves time limits to get the patient out of shock.  If the time limit is not met more aggressive measures need to be started.
  5. Harwood comment: I agree the ABG is an underused tool for figuring out the cause of sepsis.  Rechecking the HGB is important.   If the HGB is falling below 8 in a patient with and elevated troponin, transfuse.
  6. Harwood comment: If you have a sick patient with an infiltrate/pleural effusion and on the same side the patient has a small apical pneumo (air in the hemithorax) you need to consider empyema.

 

Bedside ultrasound is a good way to identify if patient has an empyema.

 

 

*U/S of empyema

 

Empyema needs to be drained. Think of it as an abscess in the chest, or similar to an obstructed/infected kidney.   All these pus collections need emergent drainage.

 

4 reasons to intubate

  1. Protect airway
  2. Failure to oxygenate
  3. Failure to ventilate
  4. Ease the work of breathing

 

Bottom line if you see a patient with pneumonia and pleural effusion do a CT or bedside ultrasound to differentiate empyema from para-pneumonic effusion.  If you identify empyema,  thoracentesis or chest tube to drain the empyema is indicated. 

 

 

Burt    Acid/Base 4 Step Method of Analyzing an ABG

 

Step 1.   Identify abnormal values on the ABG

 

Step 2. Calculate the anion gap

Na - (Bicarb + chloride)= Anion Gap

 

Step 3. Apply the Rule of 15

Bicarb +15 should =  the PCO2 and Last two digits of the pH.   This step will identify a “hidden respiratory” process.    As an example if the PCO2 is higher than expected by the rule, there is a respiratory acidosis in addition to the primary metabolic acidosis.  If the PCO2 is lower than predicted by the rule, there is a respiratory alkalosis.

ABG:  ph=7.1, PO2=200, PCO2=30, HCO3=10.   FIO2=100%  O2 on vent

Bicarb  +15 = 25.  Based on the rule of 15 the pH should be 7.25 and the PCO2 should be 25.  However the measured pH is 7.1 and the PCO2 is 30.  Because the PCO2 is higher than predicted by the rule there has to be a respiratory acidosis in addition to the metabolic acidosis. 

 

Step 4.  Calculate the Delta Gap

Subtract  the delta gap (AG – 15) from the normal expected bicarb (24) to identify the predicted bicarb. 

If  actual measured bicarb is higher than predicted then there is a hidden metabolic alkalosis.   If the actual measured bicarb is lower than predicted then there is a hidden metabolic acidosis.

Example:  Anion Gap =25,  Hco3 = 15.    25-15=10.    24-10=14.  The actual measured bicarb (15) is pretty close to the predicted bicarb (14) so there is no “hidden” metabolic process.

Example 2: Anion Gap= 30, HCO3= 20    30 -15 = 15 .   24 -15 =9    The predicted bicarb  (9) is less than the measured bicarb 30 indicating a “hidden” metabolic alkalosis in addition to the metabolic acidosis.

 

Elise comment: The delta Gap calculations (Step 4) are difficult and rarely will make a clinical difference in the ED. You probably can get by with the first 3 steps alone. 

 

VBG’s and ABG’s have reasonably similar values for ph, pco2, and bicarb.  The PO2 is the value that is significantly different between an ABG and VBG.

 

*ABG  vs VBG

 

 

Parker     ACS

 

 

*STEMI vs NSTEMI

 

 

*Posterior EKG

 

 

* Antero- Lateral STEMI

 

 

*Inferior STEMI

 

*RV Infarct (III with more ST elevation than II.  Also ST elevation in V1)

 

Inferior STEMI’s tend to have bradycardia and blocks.  They can have papillary muscle rupture.   Overall inferior STEMI’s tend to do well.

 

Anterior STEMI’s  tend to develop CHF, shock, wall rupture, and malignant arrhythmias.   Anterior STEMI’s have higher mortality than inferior STEMI’s

 

Right ventricular MI’s are very volume dependent and can develop hypotension.  Be very cautious with IV NTG.  Give fluid boluses to support pressure.

 

Treatment of STEMI’s :   ASA,  Nitroglycerin,  Heparin are the basics. 

 

 

*Sgarbossa Criteria for diagnosing AMI with LBBB or Paced EKG

 

Myers       Administrative Lecture

 

EM Doctors and nurses should meet and update the patient together prior to discharge or admission.

 

Be careful when ordering cervical/vaginal diagnostic studies.   Frequently the wrong swabs are sent.  Be careful to label your specimens. On the Label, note the time you actually obtained the sample.  Samples need to be sent to lab within one hour of being obtained.  If the test was ordered initially but then later you decided not to gather samples, cancel the order. When obtaining GC/Chlamydia samples use the blue swab only.  The current CDC recommendations state you can swab anywhere in the vaginal vault to obtain GC/Clamydia samples.  You don’t have to swab the cervix.  Give the patient the time frame of 7-10 days for GC/Chlamydia results.

 

If you are swabbing an extra-genital site for GC/Clamydia you have to use an E-swab for GC culture and UTM  (universal transport media/influenza medium) for chlamydia culture.  You don’t use the GC/Chlamydia swabs we use for vaginal/urethral testing on extra-genital sites.

 

Medication reconciliation is critical to patient safety.  A good resource for patients’ medications is in Firstnet.   Under Orders, click on “external medication references.”  When the window opens up, you may have to click on refresh multiple times to get the list.  You can also click on “medication list” under Orders in Firstnet.  Use other resources as well: patient, family, nursing home records, prior records, and primary physician.  

Elise and Kelly comment: It is the physician’s responsibility to identify the medications the patient is on that could be possibly causing the acute condition.  It’s not the physician’s responsibility to put the list into the computer, but in the end we are responsible for identifying critical meds like warfarin or rivaroxaban.

 

 

 

Conference Notes 6-3-2015

If you don't see images scroll to the bottom an click on "view in browser"

Gore    M&M

Elderly female presents from the nursing home with leg wounds.  Patient was in the hallway and generally not that sick appearing.   However technically, she had SIRS criteria based on heart rate of 95, RR=22, and temp of 38.1 .   Labs showed a lactate of 7.3.   Patient had bands on her CBC as well as toxic granulocytes.  The patient’s  appearance and location in the hallway belied the fact that she was actually in severe sepsis.

 

*SIRS Criteria 

 

Surgery consultants did not feel that the patient’s leg wounds required surgical debridement.

 

Sayger comment: General surgery lately has been pushing back on cases outside of the abdomen.  Other conference attendees noted an impression that there is sometimes disagreement by general and specialty surgical services  about which service has primary responsiblity for some body areas.  Elise comment: These disagreements need to go up the food chain to involve the Departmental Chairs and Administration.

 

The patient developed hypotension.  Fluid resuscitation resulted in decreased O2 sat and bilateral rales in lungs.  It was later learned that patient had a history of diastolic heart failure.   At this point sign out occurred. 

The sign out team placed a central line for pressor support.

Patient was admitted to ICU.  Blood cultures grew out strep.   Consultants generally agreed that the wounds were chronic and did not require debridement or amputation.

Christine comment: The leg wounds may have been a portal of entry for strep bacteremia/sepsis but they may have not been infected per se.

 

Faculty discussion about the cognitive biases we face with ED patients, especially hallway patients.  Humans have two cognitive systems at work in their brains.  System 1 is fast, intuitive, and relatively effortless. That is the system we depend on the majority of the time.  System 2 is analytical but slow, lazy, and effort intensive. It lags behind system 1.   As ER docs we have to make sure our system 1 (intuitive) is built on good knowledge of decision rules like the HEART score or PERC rule.  Then we neeed to take some time with our cases and seek out other cognitive cues to give our system 2 time to do it’s job to analyze the clinical situations we face.   To read more on this topic I would suggest the book, “Thinking Fast and Thinking Slow” by Daniel Kahneman

 

*Thinking Fast and Thinking Slow

 

 

Knight   Bites and Antibiotics

 

5% of untreated dog bites and 80% of untreated cat bites will get infected.   Treat with Augmentin for both if the patient is not penicillin allergic.  Elise comment:  Augmentin for bite wounds is known as “Dogmentin”

For pen allergic patients you can give clindamycin/floroquinalone dog and human bites or clindamycin/doxycycline for cat bites.  For pen allergic kids you can give Zithromax.

 

High risk wounds: All cat bites, deep dog puncture wounds, hand wounds, and immunocompromised patients, and any injury going to the OR are considered high risk.

 

Cat scratch disease results in regional lymphadenopathy 7-12 days after a cat scratch.  It is due to Bartonella infection. Antibiotics are not indicated unless adenopathy is painful or the patient is immunocompromised.  If you treat, 5 days of azithromycin is reasonable.   Elise comment: Kitten scratches are more prone to cat scratch disease than cat scratches.

 

Treat all human bite wounds as infected.   Fight bite wounds are high risk.  You should give at least one dose of IV antibiotics, check an xray, splint the injured joints, and consult hand surgery for these cases.   Human bite wounds are polymicrobial infections and may also have eikenella.  Augmentin is again a good choice for these wounds.  For pen allergic patients you can use clindamycine and a flouraquinalone.

 

*Fight Bite

 

Bites from rats, mice, and squirrels are also treated with Augmentin. 

Rat bite fever: Rigors, fever, polyarthralgia.  Mortality 10%.  Treat with Augmentin/Unasyn.

Livestock bites have more potential for tissue damage and systemic infection. 

 

Rabies is a risk for bites from dogs, bats, monkeys, skunks, raccoons, and foxes.  Rabies is not a risk for rats, mice, squirrels.

When giving HRIG, infiltrate around the wound and give the remaining HRIG IM at a site away from the vaccine site. 

HDCVaccine is given at days 0,3,7,and 14.

Elise comment: Small animals like rats don’t carry rabies because they have a high metabolism and die before they can infect someone.  However, groundhogs are an example of a lagamorph/rodent  that can cause rabies.

Cirone comment: There was a case of rabies in cattle in Illinois in 2011. 

Elise comment: Bats are the most common rabid animal in Illinois

Faculty consensus: A patient bit by your average family dog in cook county does not need rabies prophylaxis.  If the dog has been exhibiting aggressive behavior or unusual behavior or it was one of a pack of feral dogs or the dog just was transported from out of the country  then give rabies prophylaxis.

 

It is recommend to suture  the following bite wounds: Face or scalp, less than 6 hours old, simple with no underlying injury, and non-immunocompromised patient.   

 

Parker      COPD Exacerbations

 

COPD Exacerbation=Acute change in baseline dyspnea, cough, or sputum production.

 

Be cautious about O2 administration.  Aim for an O2 sat of @92%.  Higher O2 saturations can diminish the patient’s respiratory drive and result in hypercapnea. In COPD, supplemental O2 should raise the O2 sat relatively easily.  If you need a lot of O2 to raise the O2 sat start thinking about alternative diagnoses like PE or pneumoia.

 

Give antibiotics for admitted patients, increased sputum purulence, increased dyspnea, and bipap’d or ventilated patients.  Basically this will include most patients and definitely any patient that is a little bit sicker.  In general you can use a macrolide, flouroquinolone, or doxycycline.  For severely ill patients be sure to cover pseudomonas with Levoquin, Cipro, or anti-pseudomonal cephalosporins. 

 

Magnesium is cheap and safe.  Give 2gm IV in severe cases.  A recent study for asthma (not COPD) showed no benefit of magnesium in asthma so extrapolating to COPD suggests that it may not be of benefit.   Motzny comment:  Magnesium may have some respiratory parameter benefits (disease oriented end points) but no patient oriented outcome benefits have been identified (no improved mortality, lowered intubation, shorter stay, etc)

 

Bipap has been shown to decrease intubations by 65% in COPD patients.

When intubating, bag with a low rate to allow time to for the patient to expire trapped air.  The best ventilation strategy would a be low rate (10 or les),  I:E ratio of 1:5, assist control,  tidal volume of 6 ml/kg, 100% initial FIO2, and peep of 5 or less.  Keep plateau pressures <30.  Following the peak pressure is not clinically helpful.   Ventilate with permissive hypercapnea. 

 

You can use manual decompression (take the patient off the ventilator and manually compress chest to force air out) on an hourly rate to increase air movement out of the chest.

 

Harwood/Girzadas comment: Use a lung protective strategy with TV of 6ml/kg, Rate of 8-10, and start with FIO2 of 100% and titrate down to an O2 sat of about 92%.   Braden comment: If you keep the O2 sat higher you may benefit by knocking out the patient’s respiratory drive and allow the vent to more completely control the ventilations. 

 

High flow, High humidity nasal cannula can deliver 40L/min of O2 with added humidity.  It opens up alveoli and provides positive airway pressure.   Girzadas comment: Will high FIO2 drive up the O2 sat and decrease respiratory drive?  Elise and Bradon response:  You can set the flow rate, humidity rate, and FIO2 so you can use this modality and set the FIO2 at 40% or something other than 80-100% FIO2 and not knock out the patient’s respiratory drive.   Febbo comment: This will be an important modality for hypoxic lung problems like pneumonia maybe more so than COPD.

Barounis comment:  High flow O2 gives the patient some peep (3-5). It stents open the airway.   You should use this when intubating a hypoxic patient.   Elise comment: When is bipap a benefit over high flow O2?  Barounis response: CHF is probably the time to use bipap.   It does provide more peep than high flow O2.   Purely hypoxic patients like PE’s and pneumonia’s will be better served by initially trying high flow O2 than bipap.

 

Barounis recommendations:

Bipap for CHF and COPD

Hi Flow O2 for hypoxic pneumonia or PE.  Use for pre-intubation.  Also consider trying this modality initially for severe asthma. 

For the patient who is struggling with the bipap mask, you can also try high flow O2.

 

Kennedy   5 Slide FU

41yo NH patient with dyspnea and tachypnea.   Patient is febrile and tachycardic.  Patient’s limbs are diffusely rigid.   CBC with 26.7 WBC’s.  Lactate is 5.1.   Initial CK was 253.  CSF was clear with 2 wbc’s.  UA showed wbc’s but no bacteria. 

 

Most likely diagnosis was neuroleptic malignant syndrome vs. sepsis. 

 

*Neuroleptic malignant syndrome

 

*Neuroleptic malignant syndrome DDX

 

Patient’s CPK increased markedly.  Neuroleptic malignant syndrome was diagnosed.

 

Treatment for neuroleptic malignant syndrome: Correct hyperthermia, IV fluids, give benzo’s and dantrolene for muscle relaxation. 

Andrea comment: If there are no serotonin meds on board, give bromocriptine as well.  Bromocriptine can worsen serotonin syndrome so avoid this medication in any cases where serotonin syndrome is a possibility (serotonergic meds on board). 

 

Toerne      Acute ETOH Withdrawl

 

Case 1.  63 yo male with hx of hypertension.  He drank a pint of vodka daily.  Presented with tachycardia and hypertension. Patient had a facial contusion and a left hand injury.  Magnesium was low and LFT’s were mildly elevated. Head CT was normal.

The DSM5 uses the term Alcohol Use Disorder as a broad term that encompasses alcohol abuse and alcoholism.

Chronic ETOH use results in an extrinsic tonic sedative effect on the brain.  The brain compensates by increasing excitatory activity.  This compensatory response includes glutamate release and up regulation of NMDA receptors.  GABA receptors are down-regulated and there is decreased sensitivity to benzo’s.   Then when the compensated patient ceases ETOH use, you get a brain that is hyper-stimulated. 

 

4 ETOH Withdrawal Syndromes

 

  1. Seizures:  Brief tonic clonic seizures early in the withdrawal state.  <3%develop status epilepticus.  40% will go on to DT’s.
  2. Uncomplicated Alcohol Withdrawal: 6-24 hours after cessation of drinking. They have tremor, tachycardia, diaphoresis but no altered mental status.
  3. Hallucinosis: Ted believes this diagnosis does not actually exist. Hallucinations in this syndrome are not associated with disorientation.  Very few cases of this has not been reported.
  4. Delerium Tremens: Starts 2-4 days after cessation of alcohol use.  The earlier DT’s occur following alcohol cessation portends worsening symptoms/signs.  The hallmark of DT’s is disorientation, global confusion, and hallucinations.   As with other forms of delirium, DT’s encephalopthy will wax and wane.   The patient also will have peripheral effects of a hyperadrenergic state.   Be sure to rule out other causes of delirium.  You can have DT’s without prior seizures.  Seizures and DT’s are separate entities.  They can go together or not.

 

There was a group discussion regarding the example of a patient who normally lives at the 500-600 range of serum ETOH levels.  When that person drops to the 250 range they develops DT’s.   There was consensus that “relative ETOH deprivation” can result in alcohol withdrawal syndromes even with relatively high serum ETOH levels.

 

For Ted the key question to ask a patient is “How many days per week do you drink?”  Then “When you drink how much do you drink?”  Patients will minimize their alcohol use.  One way to get at how much ETOH a patient drinks is to ask how much alcohol the patient buys. 

Clinical cues to occult alcohol abuse: swollen hands and feet, hypomagnesemia, mild LFT elevation, mild macrocytic anemia, mild thrombocytopenia.

 

Criteria for admission: Significant medical diagnosis, significant trauma diagnosis, and severe uncontrolled withdrawal symptoms. Seizure may or may not indicate need for admission.

 

Management: Provide a quiet environment,  frequent re-assessments of the patient, evaluate for other medical problems. 

Give medications to get the patient to a point where they look like they are sleeping. 

 

High dose lorazepam >10-15mg/hr can result in lactic acidosis due to the propylene glycol in lorazepam.  Lower dosing regimens don’t cause lactic acidosis.

Febbo comment: To get rapid control of a patient having severe withdrawal symptoms, IV valium may be a better choice than lorazepam due to it’s rapid peak action at 5 minutes.  Ted agreed with this.

 

Phenobarbital acts synergistically with lorazepam.  It is cheap and long acting.  The combination of phenobarbital and lorazepam can increase the risk of respiratory depression and hypotension.

Ketamine is an option because it is an NMDA receptor agonist.   Ted feels this is a great choice for rapidly sedating a very agitated ETOH withdrawal patient.  He gives 0.5 mg/kg as the initial dose.

 

Dexmedetomidine has not shown a shortened ICU stay despite resulting in overall lower benzo doses.

 

Ted’s approach/New ED Protocol for Alcohol Withdrawl:

Lorazepam  2mg, 4mg, 8mg, escalating Q 10 minutes.  If not controlled,

Lorazepam 5mg/hr continuous infusion.  If not controlled,

Phenobarbital 10mg/kg IVPB over 30 minutes.  If not controlled,

Start a phenobarbital continuous infusion.  If not controlled,

Start a ketamine infusion.

 

Elise and Harwood were uncomfortable with starting at 2mg dosing of Lorazepam.  They wanted the first dose of lorazepam to be 4 mg.

 

Katiyar        EM Billing and Coding   Increasing Procedure RVU’s

 

Be sure to write a procedure note for any procedure, even removing a crayon from a kid’s note.   Document whether you used sedation for the procedure.  Sedation for procedures increases the RVU’s for that procedure.

For burns, in addition to your H&P, document the procedure of cleansing or debridement, as well as applying antibiotic ointment and a dressing.  You can bill for any burn management procedures if you do it.

 

Document cardiac monitoring before and after treatment.  This increases your RVU’s.  Note the rate and rhythm.  Document the rate and rhythm before and after any intervention.

 

Document your assessment of the pulse ox value (adequate oxygenation, hypoxia).  Document if any intervention you took based on that assessment.

 

Document post splint care. This increases RVU’s.   You have to write something like: Neurovascular status evaluated after splint applied. Patient has normal pulse and capillary refill.  Patient has normal movement/sensation of fingers.  Splint is comfortable for patient and not overly tight.

 

Be careful with voice dictation. There tends to be a lot of errors.  You have to review your dictation for errors.  Dictation errors can be embarrassing and problematic if your chart is reviewed for medico legal reasons.

 

Length and location of the wound determines the reimbursement.  Measure each wound with the tape measure that is wrapped around the 4X4 gauze in the suture kit.  Don’t guesstimate the laceration length.  Guesstimation can under or over-estimate the length.   Digital and regional blocks also increase RVU’s.  Document them in your procedure note if you do one.

 

Optimize your diagnoses because it improves your RVU’s and improves the hospital case mix index.  For example, Acute STEMI demonstrates severity of illness better than Acute Chest Pain.  DKA is better than hyperglycemia.  Hospital reimbursement improves with a higher case mix index.

 

We can bill for smoking cessation counseling if you discuss with the patient and or family for more than 3 minutes.  You need to document your conversation and the length of time of the conversation in the chart.

Conference Notes 5-27-2015

 

Permar      STEMI Conference

Case 1.  Sorry I missed this first case

 

Case 2.  41 yo male with no prior cardiovascular diagnoses presents with chest pain and diaphoresis.  In the ED the patient was hypotensive.  EKG showed an inferior/posterior MI.   A Code STEMI was called immediately.

 

*Inferior-Posterior MI

 

Patient developed torsades in the ED.  He was resuscitated multiple times.  He made it to the cath lab.    Cath showed severe multi-vessel disease.  Despite a balloon pump, ICD, medical resuscitation, and angioplasty the patient died.  

 

Dr. Dia comments: Using a larger volume balloon in the balloon pump (50ml vs 40 ml) is more effective.   There are also temporary ventricular assist devices or ECMO that could be useful is these situations.  Balloon pump by itself has not been shown to improve 30 day mortality.

Dr. Levin and Silverman comments: There are Impella devices that could be possibly placed in the cath lab.  These are percutaneously placed left ventricular assist devices.

Dia comment: To support a cardiac arrest patient with ECMO it has to be VA ECMO so that you are supporting the left heart not just the right heart.

Harwood comment: This patient’s anatomy was fatal.  Be very careful with vasodilators (nitro) in patients with inferior/posterior MI’s. Think RV infarct.  Also intubate these very sick patients prior to sending them to the cath lab. The cardiologists agreed with these comments.

Silverman comment: I have developed a lower threshold to intubate patients prior to cath.  If we have to intubate during the cath, it slows down and interrupts the cath procedure.

Levin comment: Intubation in the cath lab takes more time than in the ED. Patient selection for these unstable patients is critical for making the best decision whether to go to cath or not .

Group discussion regarding taking post-arrest patients to cath lab. There is an EM push to get post-arrest patients to the cath lab.  Cardiology has the some contrary concerns that post-arrest patients frequently die in the cath lab or shortly therafter.  Careful patient selection and good communication between EM and Cardiology is crucial.

 

Case 3. 55yo male had out of hospital V-Fib Arrest.  Patient was resuscitated pre-hospital.  ED EKG showed an anterior MI. 

  

*Anterior MI

 

Silverman comment: This patient was continuously unstable in the ED with repeated V-Fib arrests.  He should go to the cath lab, but he was never stabilized long enough to make cardiac cath possible.  He recommended loading the patient with amiodarone and if you can get a period of time with a blood pressure present, get him to the cath lab.

 

Be sure you document you cath lab decision making.  Document that the patient is unstable, that the case is complex.  This will take them out of the 90 minute timing issues.

 

This patient eventually stabilized for cath lab. He had a 100% LAD occlusion. He was placed on balloon pump.   Patient was transferred to MICU and had another V-fib arrest.  He was resuscitated but then had PEA and died.

Dia comment:  PEA suggests to me either a respiratory problem or ventricular rupture.  Silverman comment: He had respiratory failure due to pump failure and worsening pulmonary edema.  We are moving toward having an ECMO device or Impella device to support these patients beyond a balloon pump. Cooling the post-arrest patient is also super-important. We can place a cooling catheter in the cath lab if necessary.

Harwood comment: Cooling is not indicated for patients who are conscious after arrest.

 

Case 4.  64 yo Patient presents with chest pain and subtle EKG suggesting posterior mi.  Dia comment that with AVR showing subtle elevation and diffuse mild ST depression also consider LAD lesion.  Harwood noted prolonged qt interval as well.

Posterior EKG showed very subtle st elevation in V7-9.  There was no consensus in the audience about calling the STEMI based on these EKG’s.

Patient was managed medically.  He went to cath 1-2 days later. Cath showed severe multi-vessel disease with LAD and marginal branch with 90% occlusions.

Silverman comment: This patient had very elevated blood pressure so there is a lot of room to medically manage this patient with nitrates and beta blockers and cool them down without emergent cath.

 

Cardiology comment: In the initial EKG the anterior t waves are inverted suggesting anterior ischemia rather than posterior mi which may have st depression with upright t waves.

 

*Anterior ischemia

 

*Posterior MI

 

Kadar/Putmann     Oral Boards

 

Case 1.  26 yo female with abdominal pain for 5 days.  Patient has fever and tachycardia.  History reveals vaginal discharge.  Only medication is an oral contraceptive.   Physical exam demonstrates lower abdominal tenderness and CMT and vaginal discharge.  Diagnosis is PID with a TOA.  Critical actions:  Get a pregnancy test,  provide pain control, obtain cervical and vaginal specimens, get a pelvic ultrasound, start IV antibiotics (IV cefoxitin and IV doxycycline plus IV flagyl), Consult GYN.

 

Case 2.  2 year old child with fever and rash. Heart rate/pulse/respirations are normal.  Dexi is 100. Rectal temp is 39.  History reveals 5-6 days of fever and now has a diffuse erythematous rash.  Review of systems is negative.  Exam shows red cracked lips and bilat conjunctivitis.  Patient has bilat hand swelling. There is cervical lymphadenopathy.  Skin shows a diffuse erythematous rash.  Diagnosis is Kawasaki’s disease.  Critical actions are Starting ASA and IVIG. Order ECHO and consult ID or rheumatology.   Rash can be very nonspecific. 20% of untreated patients will develop coronary artery aneurysms 2-6 weeks after illness.  These aneurysms can lead to sudden cardiac death later in life.

 

*Kawasaki’s

 

Case 3.  3 year old with possible nasal FB. Vitals are normal.  Airway is patent, no respiratory distress or choking.  Mom states the child said he put something in his nose and has pain. Exam demonstrates FB in the left nostril.  Critical actions: Remove FB with kissing technique (mom blows air into child’s mouth rapidly with non-affected nostril occluded) or forceps or balloon catheter technique.

 

McVicker  (312-550-1941 You can text Tricia with any safety event)  

Root Cause Analysis

 

No one goes into healthcare to hurt patients or break rules.  If an error happens it is because our system broke down.

 

Root Cause Analysis is a structured process for identifying the causal or contributing factors underlying adverse events or other critical incidents.

 

Accidents in health care almost never stem from a single cause. They come from a mix of failures or work conditions that align precisely to slip through every existing defense.

 

 

*Swiss Cheese Model of Error

 

The goal of RCA’s is to learn from adverse events and prevent them from happening in the future.  We talk about what happened and why it happened.  Then we try to come up with a plan to prevent it from happening again.

 

Residents are very helpful to the RCA process because they know the micro processes of patient care.

 

ACMC does RCA’s when there is an event of unexpected death or severe harm to a patient.  RCA makes safety a real priority and focuses on how the system failed.  It does not focus on personal blame.

 

The RCA process is protected under the Illinois Review Act.  It is not discoverable for malpractice cases.

 

The RCA process identifies the probable causes of an event, looks for reasons why they happened, and then identifies ways to prevent such an event from happening again.

During the RCA a timeline of events is developed.  Then causal events (the why’s) are mapped out in a flow chart.

 

Cash  M&M

 

“Failure is instructive”  “We do not learn from experience…we learn from reflecting on our experiences.”

 

Case 1. 35 yo female with hyperglycemia and LLQ pain.  Ct showed ureteral stone.  UA showed blood and WBC’s. 18 hours later, patient was febrile and tachycardic.  Creatinine had bumped to 2.58.  Patient became hypotensive.  IV antibiotics were started.  Nephrostomy tube was placed.    Patient later expired from sepsis.

 

Reflections: Take ownership of sign outs. They are truly your patients.  Don’t slip into autopilot mode.  Make sure that planned interventions are getting done.

Girzadas comment: Keep a lookout for these obstructed kidneys with any signs of infection (fever, wbc’s in the urine).  There is urgency in getting these kidneys drained by IR with a nephrostomy tube.

 

Case 2. 30 yo female had left flank pain.  UA shows leukocytes and blood. Ultrasound shows left renal hydronephrosis.   Vitals show HR=108 and BP =106/72.  Patient was discharged.    2 hours later the patient returns to the ER unresponsive.  Vitals were 39.8C and hypotensive/tachycardic.   At this point lactate is 7.  IV Fluids, IV antibiotics were given and patient was intubated.  An emergent nephrostomy tube was placed.  Urine showed klebsiella.  Patient improved and was discharged home.

 

Reflections: Pay attention to vital signs.  Beware of communication failures between nurses and docs.  Be vigilant with hallway patients and on busy shifts.

Harwood comment: The receiving sign out team has fresher brains and should be willing to re-evaluate the patient later during their shift.

Natalie Htet comment: Using ultrasound can show hydronephrosis in these patients but I have seen two patients who had some hydro seen on ultrasound come back to the ED and large ureteral stones were then identified on CT.  Hydro on ultrasound does not give you much data on the size of stone or severity of obstruction.

 

Case 3.  65 yo female with abdominal pain and fever.  Tachycardia and hypotension.  Patient notes some dark urine.  On exam patient has suprapubic tenderness and mild right CVA tenderness.  Urine shows blood and leuks.  Diagnosis is pyelonephritis.  A Ct was performed.  Patient had an obstructing stone in right ureter. Right nephrostomy drainage tube was placed.  Urine culture shows Klebsiella. 

 

There was a discussion concerning whether we should be doing ultrasounds or CT on pyelonephritis more frequently to rule out ureteral stone/obstructed kidney.  Mila Felder suggested we review a series of cases to develop a more robust clinical pathway for pyelonephritis.

 

Sola comment: I have been burned on signouts that say  “This patient is being discharged so we don’t need to talk about this patient”  A patient planned for discharge started vomiting and needed further care and I knew nothing about the patient.  Harwood comment: I even ask about the patients that died on the last shift.  In case family later shows up.

 

Ede     5 Slide Follow Up

 

27 yo female with RLQ abdominal pain and syncope.   UCG was positive.  Pain radiated to right shoulder.  Exam shows BP 107/74   HR=95.  Minimal blood in vaginal vault. Cervix closed.  UA shows large blood.  HGB=11.6.   Beta HCG= 67.  U/S showed free fluid in RUQ.   Patient was taken to the OR.  A left tubal pregnancy was removed.   The patient had a ruptured ectopic pregnancy with a Beta HCG of 67. 

 

Risk factors for ectopic pregnancy: PID prior tubal surgery, IUD, assisted reproduction, and previous ectopic.

 

Triad of ectopic pregnancy: Abdominal pain, vaginal bleeding, and amenorrhea.

 

Findings suggestive of ectopic on U/S: free fluid, mass, no intrauterine pregnancy.

Beware of heterotopic pregnancy in patients who are undergoing assisted reproduction therapy.

Harwood comment: Ectopic pregnancies are reported/known to have low Beta HCG’s.  So don’t let a low Beta HCG lull you into complacency.

 

Munoz    5 Slide F/U

 

4 yo female with abdominal pain and lethargy.  Patient had a history of Graves disease and is on propranolol.  Child had been sleeping all day.   Vitals show tachycardia and tachypnea. No fever.  Child was lethargic on exam.  Stool and emesis were heme positive.  Labs significant for anion gap acidosis with an osmolal gap.  ED docs considered toxic alcohol ingestion. Patient was admitted to PICU and eventually was diagnosed with thyroid storm.  7 days later the patient had thyroidectomy.

 

*Burch-Wartofsky Criteria

 

*Treatment for Thyroid Storm

 

Harwood comment: This seems like a case of apathetic thyroid storm in a child.  This could be a case report material.     

 

 

 

 

 

 

 

 

 

Conference Notes 5-20-2015

If you don't see images scroll to the bottom and click "view in browser"

Ryan      Musculoskeletal Emergencies   Study Guide

 

Spinal stenosis clues: patient walks in flexed forward position.  Patient has lower extremitypain when walking.  Patient has pain relief with sitting.  No pain with sitting is the most specific historical clue.  Spinal stenosis causes pain with exertion similar to vascular claudication.  However, spinal stenosis pain does not resolve when the patient stops walking and remains standing.  Vascular claudication should resolve when the patient stops walking and remains standing. Spinal stenosis pain won’t resolve until the patient can flex his hips by sitting , leaning forward, or squatting.

 

*Spinal stenosis

 

*Normal healthy spine

 

Adequate muscle relaxation is the most important factor in successful shoulder reduction

 

 

*Testing for supraspinatus Empty Beer Can Test (Jobe’s Test) Pain and weakness resisting downward pressure on the arm suggests a tear in the Supraspinatus Tendon

 

*Jobe's Test

Harwood comment: For knee injuries that swell immediately, there probably is a hemarthrosis.  That is the only mechanism for a knee to swell immediately.  For knees that swell up overnight that is more likely a reactive effusion.

 

Cauda Equina syndrome is associated with urinary retention.

 

*Cauda equine syndrome

 

*U/S of urinary retention

 

Reimplantation of amputated digits is basically all about the thumb.  Discusss with the hand surgeon prior to discarding any tissue.  Keep the amputated finger wrapped in saline soaked gauze.  Place the saline gauze-wrapped digit in a plastic bag and place the bag on ice. Keep the digit in a cooler that stays with the patient.

 

 

*Gout vs Septic Arthritis

 

Jeziorkowski     Trauma to Cervical Spine

 

*Nexus C-spine Rule

 

*Canadian C-Spine Rule

 

Elise feels the Canadian Cspine rule is superior to the Nexus rule based on it’s increased specificity. If the Canadian C-Spine rule indicates imaging, go to CT rather than plain films.

We also discussed the practice of placing an axial load on the patient’s head to check for tenderness.  It was felt among the group that if you do the Canadian Cspine rule first with head turning prior to axial loading, it probably isn’t a dangerous practice to axial load, but it also is not useful.  Axial loading is probably not a useful test for assessing the Cspine.

EAST recommends CT as the imaging method of choice for the Cspine.  Plain films are only indicated for a young, non-obese, non-extremely muscular person with a low-risk mechanism of injury.

 

Chris discussed multiple unstable C-spine fractures

 

 

*Unstable Cspine Fractures

 

*Cspine Fractures

 

*Flexion Teardrop Fracture

 

*Hangman’s Fracture

 

 

If CT Cspine is negative and patient has persistent severe pain or paresthesias you need to get an MRI.  There is an entity called SCIWORA, which is a spinal cord injury without radiographic abnormality.   This injury is usually a ligamentous tear.  This injury can be best picked up by MRI.  There was consensus among the attendings that flex-ex films are inferior to MRI for picking up ligamentous injury.  Also all the attendings felt that patients who have some persistent midline posterior neck pain following a negative CT cspine but have low to moderate-risk mechanism and no neuro symptoms can go home without an MRI.  It should be said that EAST still recommnends MRI for patients with persistent pain following negative CT. Most attendings felt it was unnecessary to MRI all these patients.  A selective approach based on severity of pain, mechanism, and neuro symptoms seems reasonable.

 

EAST Guidelines recommend removal of c-collar after a negative CT scan of Cspine in a comatose patient. This is a brand new recommendation.  There was some collective head scratching on this recommendation by the faculty. Most faculty felt they would still MRI this type of patient (head injured, comatose, in c-collar following negative c-spine CT)

 

Cash/Lovell      Clinical Teaching

 

 

*One Minute Preceptor.   Getting a learner to commit to a diagnosis they feel is most likely and defend that commitment is a key to this teaching method.

 

SPIT  teaching method:  Ask the learner to give a differential diagnosis in each of the following SPIT categories

Serious

Probable

Interesting

Treatable

This is an easy way to teach on almost every patient.

 

 

Create a motivating environment.  Be an encouraging and supportive teacher.  Give the learner specific feedback.

 

 

3 Step approach to teaching procedures

  1. Assess your learner’s skill level
  2. Teach Concepts.  This includes repetition of the tasks of the procedure.  Introducing a small bit of variation regarding how to perform the procedure helps with performance and long term retention of the skill.  With more advanced learners’, focus on key portions of the procedure and discuss how to refine their technique.  “What if” simulation scenarios a la Harwood asking you how would handle this procedure if the patient was pregnant or was a pediatric patient or had septic shock can improve the advanced student’s learning.
  3. Directed Feedback.  Be specific in your analysis and suggestions to the learner.

 

 

*Deliberate Practice

 

 

*Peyton’s 4 Step Model

 

*Elise then showed a video of how to make rabbit ears based on Peyton’s 4 Step Model.  This skill was clearly mastered by multiple residents

 

Jamieson/Watts    Oral Boards

 

Case 1.  Middle aged man presents with syncope while watching the Hawks game.  Patient has history of cold symptoms for 2 days. Physical exam is unremarkable except for URI findings.  EKG shows brugada changes. 

 

*Brugada EKG V1-3 has RBBB pattern with ST elevation

 

*Brugada types

 

Critical actions: Consult cardiology and admit for AICD placement.

 

Case 2.  16 yo female with hematemesis and syncope.  Patient is tachycardic.  Patient has history of autoimmune hepatitis.  Patient develops massive hematemesis in the ED and becomes unresponsive.  Critical actions: Patient is intubated to protect airway. 2 large bore IV’s, IV fluid resuscitation, IV PRBC transfusion. Protonix and Octreotide.   Consult GI and admit to ICU.  Ideal management includes NG tube and IV antibiotics (Rocephin or a Quinalone).

 

(International Journal of Hepatology 2011) Currently, it is recommended that short-term antibiotic prophylaxis, a measure that reduces bacterial infections [23], variceal rebleeding, and death [24], be used in every patient with cirrhosis admitted with gastrointestinal hemorrhage [20, 25, 26]. Different antibiotics have been used in different trials compared with placebo (Table 1, [27–30]). Bacterial infection is commonly associated with variceal hemorrhage and appears to be an independent risk factor for failure to control bleeding [31] and predicts both early rebleeding and death [32, 33]. The routine use of prophylactic broad-spectrum antibiotics has shown a marked improvement in outcome in acute variceal hemorrhage. Routine intravenous ceftriaxone or postendoscopic norfloxacin reduces rebleeding rates compared to on-demand antibiotics (Table 2) [24, 29, 34–36]. A Cochrane meta-analysis of antibiotic prophylaxis in cirrhotic patients with gastrointestinal bleeding involving 12 trials with 1241 patients evaluated antibiotic prophylaxis compared with placebo or no antibiotic prophylaxis. Antibiotic prophylaxis compared with no intervention or placebo was associated with beneficial effects on mortality (RR 0.79, 95% CI 0.63 to 0.98), mortality from bacterial infections (RR 0.43, 95% CI 0.19 to 0.97), bacterial infections (RR 0.36, 95% CI 0.27 to 0.49). They concluded that prophylactic antibiotic use in patients with cirrhosis and upper gastrointestinal bleeding significantly reduced bacterial infections, and seems to have reduced all-cause mortality, bacterial infection mortality, rebleeding events, and hospitalisation length. These benefits were observed independently of the type of antibiotic used [37, 38].

 

Case 3. 26 yo female with right index finger pain. Temp is 37.9C. Other vitals are normal.  Patient has history of gardening and jabbed her right index finger on something sharp in the soil and suffered a laceration.   Her tetanus status is up to date.  Right hand demonstrates Knavel’s signs of flexor tenosynovitis on exam. Testing in the ED shows a positive pregnancy test.  Patient also has a penicillin allergy. Critical actions: IV Clindamycin (Pregnancy Class B) to cover Staph, x-rays of right hand, Hand surgery consult for surgery.

 

*Knavel’s Signs of Flexor Tenosynovitis

 

Harwood comments:  Great job telling the examiner what you know and thinking out loud.  Get a rhythm strip on the first patient ASAP don’t wait for the EKG.   You don’t need a CT scan of the head as part of a routine syncope work up.

 

Faculty felt that the tenosynovitis patient needed broad coverage including gram positive and gram negative coverage.  Elise and Harwood felt a third generation cephalosporin plus/minus clindamycin would be reasonable.  Girzadas felt clindamycin to cover MRSA was indicated.

This will give the alumni a laugh, but Girzadas advised doing a rectal exam on all syncope patients.

 

 

Lee   5 Slide Follow Up

 

35 yo female 28 weeks pregnant presents with abdominal pain. Patient was hospitalized at an outside hospital a week earlier for renal stones and ureteral stent placement.  Vitals were normal/afebrile.  On exam the patient had right lower quadrant and right flank tenderness.  CBC showed a WBC of 15.  Urine was negative for infection and positive for moderate blood.  The initial thought was that the patient was having recurrent ureteral colic. Renal ultrasound here at ACMC showed mild hydronephrosis but no stones. 

 

A pelvic ultrasound performed here at ACMC showed a right adnexal mass c/w right ovarian torsion.  Laparotomy removed a necrosed dermoid tumor.  It is conjectured that calcifications in the dermoid tumor that was adjacent to the kidney may possibly have complicated the ultrasound of the kidney performed at the outside hospital. This could have led to the initial diagnosis of nephrolithiasis and renal colic.  We however did not have those images to review.  

 

Pregnancy is associated with an increased risk of ovarian torsion.  Torsion is most likely to occur between 10-17 weeks gestation.  Torsion presents with pelvic pain, adnexal mass and nausea/vomiting.

The ovary moves upward toward the kidney as pregnancy progresses.  This may make the diagnosis more difficult.

DDX for RLQ pain in a pregnant patient =ruptured cyst, ovarian torsion, appendicitis, tubo-varian abscess.

 

*MRI demonstrating right ovarian torsion with concurrent pregnancy.  Notice how high that right ovary is.

 

 

Bamman   5 Slide Follow UP

 

3 yo male presents with fever and drooling  Fully immunized.  Child was cranky with chapped lips.  There were no intra-oral ulcerations.  The patient did not improve in the ED with magic mouthwash. And would not take a popsicle.  Because this patient looked sicker than the usual kid with stomatitis and he did not have intra-oral ulcerations, a lateral soft tissue neck film was ordered.

Lateral soft tissue neck film showed possible epiglottitis.

 

Patient was taken to the OR for intubation.  On laryngoscopy patient had epiglottitis.  There was some tracheal pus. Tracheal cultures showed staph aureus.  Patient was treated with Vancomycin and Zosyn and decadron.  Patient recovered well after 3 days of intubation.  Staph areus is becoming more common as an etiology of epiglottitis.

 

*Epiglotitis.  This patient has a thumb sign and a vallecula sign.  The vallecula sign is when the air column in the vallecula does not extend to the hyoid.

 

 

 

 

 

 

 

 

 

 

 

 

 

 

 

 

 

 

 

 

 

Conference Notes 5-13-2015

If you don't see images, scroll to the bottom and click "view in browser"

Burns/Collins    Oral Boards

 

Case 1. 27yo female passed out at home.  HR=62, BP=79/51, Afebrile.   Patient notes abdominal pain.   No PMH.  Patient is taking OCP’s only intermittently and UCG is positive.  Patient is persistently hypotensive despite IV fluid resuscitation.   FAST exam shows free fluid in Morrison’s pouch.  Diagnosis was ruptured ectopic pregnancy.  Critical actions were IV Fluid and Blood resuscitation, Rapidly getting the patient to the OR, and Giving rhogam  (pt was RH negative).

 

*Free Fluid in Morrison’s Pouch.  This finding with shock and a positive pregnancy test has a positive predictive value of >90% for ruptured ectopic pregnancy.

 

Case 2. 27 day old child with diarrhea, fever, and “not acting right.”  T=100.3, HR=160, BP=75/50  RR=46  PulseOx=96% on RA.  Decreased urine output.  Exam shows signs of dehydration.   Patient was given 20ml/kg bolus.  A septic work up including LP was begun.   LP showed 200 WBC’s.  Treatment for meningitis was begun.    Critical actions:  IV fluids, septic work-up, always consider evaluate for possible abuse, treat with antibiotics for meningitis.

 

Case 3. 38yo man brought in by his father.  Father is concerned that patient is suicidal.  Vitals are normal.   Patient refused to speak with ED physician.  Patient’s dad said that patient took an overdose of tylenol and threatened to shoot himself.   Tylenol level was 350.  Patient was treated with NAC per protocol.  Patient’s duffel bag was taken away from him.  When searched, his bag contained a handgun.  As a note to the readers, the conversation between Dr. Burns and Dr. Collins (AKA Angry Jerry) was a classic!  Dr. Burns calm, flat responses to Dr. Collins angry outbursts rivaled a comedy skit.  Critical actions: Prevent patient from leaving ED, make sure patient’s possessions are taken away from him to prevent him from using a weapon in the ED.  Obtain an acetaminophen level.  Treat with NAC.  Consult Psychiatry and Poison Control.  Admit to ICU.

 

*Rumack Matthews Nomogram

 

The approved 20 hour IV dosing regime is complicated and is performed as follows:

Administer an initial loading dose of 150 mg/kg IV over 15 to 60 minutes (we recommend 60 minutes).

Next, administer a 4 hour infusion at 12.5 mg/kg per hour IV (ie, total of 50 mg/kg over 4 hours).

Finally, administer a 16 hour infusion at 6.25 mg/kg per hour IV (ie, total of 100 mg/kg over 16 hours).

 

72 hour oral protocol — The 72 hour oral (PO) dosing protocol for N-acetylcysteine treatment has been used successfully in the United States for more than 30 years, and consists of the following:

A loading dose of 140 mg/kg PO, followed by

A dose of 70 mg/kg PO every four hours for a total of 17 doses

*NAC Treatment Protocols from Up to Date

 

Girzadas comments: Positive FAST exam in the setting of shock and positive pregnancy test has a positive predictive value of over 90% for ectopic pregnancy.  You absolutely have to LP febrile kids under one month of age.  The physical exam of kids this age is unreliable for identifying a toxic or sick/meningitic neonate.  You have to speak with family and or friends of psychiatric patients.  The psychiatric patient’s history is extremely unreliable and can mislead you to underestimate their suicidality.  You have to take the time to corroborate the patient’s history with family or friends.

There was a discussion about whether a neonate should get steroids for meningitis.  According to Up to Date: Dexamethasone is not indicated in the treatment of bacterial meningitis in infants younger than six weeks or in those with congenital or acquired abnormalities of the central nervous system.

 

Katiyar    Toxicology of Plants

 

Basic Classification of Plants

 

Alkaloids contain nitrogen.   They have bitter taste.   Most of them end in the three letters “ine”   Strychnine, Ephedrine, Morphine.

 

Glycosides  end in “in”   Digoxin, Salicin (metabolized to asprin)

 

Triterpenes     Tetracanabinol,   urushiol (poison ivy)

 

Proteins/peptides/lectins:  Ricin,  mushrooms (extremely poisonous)

 

Phenols: Capsaicin, St. John’s Wart

 

Case: 15 month has crying and drooling after chewing on a plant

 

Elephant ear plants contain calcium oxylate.  This can cause drooling, swelling of tongue and mouth.  Eye irritation.

 

Poinsettia leafs have a latex sap that can irritate mouth and throat and cause vomiting.   Poinsettias are not fatal, there have been no reported fatal cases.

 

Rhubarb leaves can cause nephrotoxicity.  Don’t eat the leaves, they can damage your kidneys and cause hypocalcemia.  

 

Jimson weed seeds if ingested can cause a significant anticholinergic toxidrome.  Jimsom weed is ubiquitous along roads.

 

*Jimson weed

 

*Salvia divinorum    Ingested leaves can cause hallucinations, coma, and memory loss.

 

Nutmeg is also hallucinogenic.   Morning glory seeds are hallucinogenic.

 

Oleander, fox glove, and lily of the valley can cause digoxin toxicity.  Patients with toxicity from these plants may need very large doses of  Digibind.  The lab testing of digoxin levels due to toxic plant ingestion can be falsely low.

 

Rhododendrons can cause hypotension, cardiac arrhythmias, and death.

 

 

*Water hemlock can cause seizures and there are case reports of death.  It looks a lot like Queen Ann’s Lace.

 

*Poison Ivy/Oak/Sumac.   These plants contain urushiol.

 

Don’t eat green tomatoes or green potatoes.  These can cause GI irritation and hallucinations.

 

Ackee fruit can cause elevated ammonia level with CNS alteration and GI distress.

 

Colchicine is contained in glory lilly and can cause multi-organ failure.

 

Ricin is extracted from the castor bean.  Jequirity pea is similar to ricin but 30 times stronger.   Both can cause hemorrhagic gastroenteritis and multi-organ failure.

 

Apple seeds in large amounts can be fatal due to cyanide toxicity.

 

Apple seeds and castor beans are not toxic if swallowed whole.

 

Bottom line to this lecture:  Don’t eat plants that aren’t sold in the vegetable section of your grocery store.   If you eat plants from other places bad things can happen.

 

Regan     Five Slide Follow Up

 

23 yo male fell while drinking and using heroin 2 days prior to ED visit .  Patient presented to the ED with left side weakness.  Patient also had left side facial swelling and left arm and leg swelling.

Lab testing showed dark urine and a serum CK of  58, 000.  Neuro work up was basically negative.  It turned out that patient had been laying on the floor for a prolonged period of time.  This resulted in rhabdomyolysis. 

 

Rhabomyolysis is diagnosed with CK level 5X normal.  Large blood in urine is a clue to rhabdomyolysis that should be followed up with a measured CK level.  CK levels>5000 increases risk of renal injury.

Treatment is aggressive IV fluids.  Alkalinize urine with bicarb  (3 amps of bicarb in 1 Liter of  D5W) .   There is no clear evidence that bicarb drip has an advantage over IV saline.

 

Orthopedic Lab

 

We had 5 stations to work on various exam and reduction techniques.

 

ICEP and SAEM Conference Pictures

 

 SAEM SONOGAMES Team  Drs Hart, Htet, Ede.

ICEP  Lindsay Purnell

 

Jennifer Cash at ICEP

Rachel Kadar at ICEP

 


Conference Notes 5-6-2015

Some further Feedback from Alumni on Peritonsillar abscess management: In my practice, the process has changed and the ENTs admit most of these patients for IV antibiotics reporting that only a small percentage require drainage.  Just FYI we may want to see what the ENT literature says on this issue...

Kristin McCabe, MD

Sent from my iPhone

Urumov           Study Guide Ortho

 

Bohler’s Angle  <25 degrees suggests calcaneal fracture.  The calcaneus is the most commonly fractured tarsal bone.   If a patient has a calcaneus fracture, look for associated fractures like hip and lumbar spinal fractures.

 

*Bohler’s Angle

 

 

*Knee Exam Tests

 

 

*Pseudo-Jones’s Fracture (Avulsion Fracture)  Treatment is with ace wrap and cast shoe and patient can bear weight.

 

*Jones Fracture   Treatment is non-weight bearing, post mold and likely surgery.

 

 

Posterior knee dislocation pretty much mandates a CT Angio looking for popliteal artery injury.    Harwood comment: In 2015, there is no reason not to do a CT Angio for posterior knee dislocations.   Prompt diagnosis of a popliteal artery injury may save an amputation.   The only reasons to not do it would be a patient with a low GFR or allergy to contrast.

 

 

*Maisonneuve Fracture

 

 

*Maisonneuve Fracture

 

 

*Osteochondritis Dessicans    Occurs most commonly in adolescents at the medial distal femur.  It is not a traumatic disease.  It is a developmental disorder.  Treated with non-weight bearing.

 

 

*Thompson’s Test for Achilles tendon rupture.    You can also use ultrasound to diagnose Achilles tendon rupture.

 

 

*US of Achilles tendon rupture

 

 

*Lis Franc Fracture   Be alert for fractures at the base of the 2nd metatarsal and  imperfect alignment of the 2nd metatarsal with the middle cuneiform.  These two findings are markers for a Lis Franc fracture.

Balogun       M&M

 

59yo male presenting with generalized weakness.  Patient has chronic jaundice that has worsened.  He uses daily ETOH.   Vitals abnormal only for tachycardia of 117.  

Patient had some bruising on his chest.  Abdomen was distended but non-tender with no peritonitis.  Liver was enlarged. Stool was brown, hemoccult positive.   Patient was tremulous.

 

Patient was initially treated with CIWA protocol, IV fluids and vitamin supplementation.

Labs showed mildly elevated troponin, mild hypokalemia and anemia to a HGB of 7.2

Ammonia level was normal.   Coagulation studies were prolonged.

Imaging showed ascites and gallbladder sludge.

 

Patient was in the ER for 6 hours.  He remained hemodynamically stable.   When patient was evaluated by the admitting physicians in the ED, the patient was less alert.

A CT done at that point, showed an intracranial hemorrhage.

 

Considering the weak cirrhotic patient, do a broad work up.  Look for infection,  elevated ammonia level, GI bleeding, anemia, intracranial bleeding, and cerebral edema.

A CT head is recommended for cirrhotic patients presenting with weakness. 

Harwood comment: Is there data to support that recommendation?

 

Elise comment:  You have to be really careful with patients who have cirrhosis.  However I am not going to CT scan the head of every weak cirrhotic patient. I will scan them if their mental status is not normal or the mental status changes in the ED.

 

There was a lively discussion about the difficulties of evaluating a patient with chronic cirrhosis and weakness.  Most people felt that weakness alone would not prompt a CT head initially.  However, these patients need frequent re-evals in the ED to evaluate for any change in mental status.   At times re-evaluation is difficult due to the pressure in the ED to keep seeing the next patient.  You have to take the time to re-evaluate the patient if a nurse calls about the patient, if an admitting doctor or consultant raises a concern, or before signout.

 

Treatment for ICH in a cirrhotic patient: FFP is the recommended reversal agent in this setting. Vitamin K has questionable benefit but you should still give it 10mg IV.  Elise comment: Don’t give Vitamin K subQ in cirrhotic patients as it can cause hematomas.   Give it IV. Give platelets if the count is <10K.  FEIBA is not recommended.  Factor 7 is not indicated. 

Harwood and Elise comments:  If the cirrhotic patient has abnormal coagulation studies but no life threatening bleed, don’t try to fix the coagulation numbers.  These patients have complex coagulation abnormalities and trying to fix the numbers may actually make them hyper-coaguable.   

 

 

Kadar      Pain Management in Acutely Injured Patients

 

Risk factors for chronic pain: high pain intensity, long duration of pain, anxiety, depression, work safety, associated litigation, poor pre-trauma health status.

 

Opiophobia:  We sometimes avoid opiates due to concerns about respiratory depression, hemodynamic instability, masking neurologic injuries, and the subjectivity of pain.    Research shows that older adults tend to get treated less for pain, many trauma patients get no pain medication and there tends to be long delays to pain medication administration in trauma patients.

The research also has shown that opioid pain medication in trauma patients does not increase intubations or other complications.

 

 

*WHO Analgesic Ladder

 

Ketamine is an NMDA receptor blocker.  It has been used by anesthesiologist to treat pain and lower the need for opioid treatment. 

 

PCA analgesia administration in trauma patients has to be used judiciously so that you don’t hinder early mobilization of these patients.   Early mobilization is key to preventing chronic pain.

 

 

Regional anesthesia has the advantage of less sedation, less respiratory depression, and less constipation/nausea.

 

 

*Facial Nerve Blocks  For longer action of anesthesia use bupivacaine with epinephrine.  Note that all three  facial blocks line up with the pupil.   Harwood comment: To anesthetize the central forehead you have to place bilateral supraorbital nerve blocks.

Elise comment:   You can lay down a line of anesthetic parallel to the upper  eyebrow to anesthetize a broad area of the forehead.

 

 

*Local Anesthetics Dosing and Duration

 

 

 

*Wrist Block

 

 

*Wrist Block

 

 

Continuous epidural infusion of lidocaine or bupivacaine can be used for multiple rib fractures, thoracic or abdominal post-operative pain.

 

 

*Intercostal nerve blocks can also be used for rib fractures.   Insert the needle in the posterior axillary line. Direct the needle down to the rib initially and then re-direct below the rib to anesthetize the nerve. 

 

 West   Follow Up  Case

 

54 yo female not feeling well a week after total knee surgery.  Patient noted some decreased urination.

Vitals show hypotension 92/31  HR=90  O2Sat=85%

Incision site did not show signs of infection.

WBC=46 with toxic granulocytes. Creatinine=3.45.  Anion Gap=26

Urine shows blood and signs of infection.

CT chest shows pneumomediastinum, free intra-abdominal air.

IV fluids were given.  IV antibiotics were given.  Surgery was emergently consulted.

Initial thought was that the patient may have an esophageal injury due to intubation from her knee surgery.  Patient did go to surgery and was found to have a perforated diverticulum and some necrotic bowel. There was no esophageal injury.

 

Central obesity, smoking, and etoh use increase risk for diverticulitis.

Patients with diverticulitis should be referred for colonoscopy in 6 weeks to be evaluated for colon cancer.  10% of diverticulitis cases can’t be differentiated from colon cancer initially.

 

There was a lively discussion about this unusual case.  It’s still hard to see the final diagnosis.  Why would a perforated diverticulum have associated necrotic bowel.  Why the severe mediastinal air?  A lot of unanswered questions. 

 

Cirone   Follow Up Case

 

Patient presents with bloody diarrhea.  Just returned from Africa.

IDPH was called when the patient arrived to the ED and IDPH advised that the patient was unlikely to have Ebola.   Patient travelled from a country that is not Liberia, Guinea, or Sierra Leon.

ID consult agreed that the patient did not have ebola.   Stool cultures were sent. 

 

*Screening of Travelers for Ebola    Screening only occurs for travelers from Liberia, Guinea, and Sierra Leon.

Elise comment: This patient came from a country that did not have  reported ebola cases.  Be careful not to raise panic in the ED for a traveler not from Liberia, Guinea, or Sierra Leon.  

Christine comment: In the middle of the night, call epidemiology because they have the most up to date knowledge of the current hospital protocols.